You are on page 1of 333
General Awareness Quantitative Aptitude j Reasoning English Language ee ee | . Head Office : 8-32, Shivalik Main Road, Malviya Nagar, New Delhi-110017 . Sales Office : B-48, Shivalik Main Road, Malviya Nagar, New Delhi-110017 Tel. : 011-26691021 / 26691713 FeO | ‘Typeset by Disha DTP Team DISHA PUBLICATION ALL RIGHTS RESERVED © Publisher No part of this publication may be reproduced in any form without prior permission of the publisher. The author and the publisher do not take any legal responsibilty for any errors or misrepresentations that might have crept in. We have tried and made our best efforts to provide accurate up-to-date information in this book. For further information about the books from DISHA, Log on to www.dishapublication.com or email to info@dishapublication.com eS Eenenaen 1" 12. SNOneON CONTENTS SECTION A : QUANTITATIVE APTITUDE Number System & Simplification Simplification Surds, Indices, Square Roots and Cube Roots Ratio, Proportion and Partnership Average, Problem on Ages Percentage, Profit & Loss Time, Work & Pipes, Cisterns Time, Speed and Distance Simple Interest & Compound Interest Elementary Mensuration Permutation, Combination & Probability Data Interpretation SECTION B : REASONING Analogy Classification Series Coding & Decoding Word Formation Blood Relation Direction, Clock and Calender Logical Venn Diagram at -10 ad -14 a15- 19 620 - 26 A27- 33, A442 043-51 852-58 059 - 64 665-74 AT5 - 82 A83 - 88 B1-6 e7-11 212-16 817-24 525-29 830-35 236-43 044-48 10. 1. 12. 13, 14, 15. hen 2 akenN Syllogism Non-Verbal Series Problem Solving Input-Output Coded Inequalities Statement & Assumptions Data Sufficiency SECTION C : ENGLISH LANGUAGE English Grammar & Vocabulary Reading Comprehension Choze Test Parajumbles SECTION D : GENERAL AWARENESS Geography Indian Polity Economy Miscellaneous Current Banking Current Affairs 249-60 261-64 265-73 5-74-84 385-92 293-100 8-101 - 108 c-1-28 0-29-50 0-51-56 57-64 b-1-10 0-11-20 0.21 -36 037-46 DAT - 54 D-55 - 68 BSE LOl ea) Number System, HCF and LCM Theten symbols 0, 1,2,3.4, represent any number, Real numbers: Real numbers comprise the full spectrum of ‘numbers. They ean tke on any form —Iractionser whole numbers, decimal points or no decimal points. The full range of eal numbers includes decimals that can go on forever and ever without end. For Example: 8,6,2+ JR. ete 5.6.7, 8,9 recalled digis, which can Natural numbers: A natural number is @ number that comes naturally, Natural Numbers are counting numbers from 1, 4,5, Whole numbers: Wholenumbers are just all the natural numbers plus zero. For Example: 0,1,2,3 5,und soon uptoinfinity tegers: Integersincerporate all the qualities of whole numbers and their opposites (or additive inverses of the whole numbers). Integers can be described as being positive and negative whole For Example: ...-3,-2,-1,0,1,2. p Rational numbers: All numbers ofthe form {> where/pand gare integers (g #0)are called Rational numbers, For Examples 4,30, 4 Irrational numbers: rational numbers arethe opposite ofrational ‘numbers, An irrational number cannot be written asa fraction, decimal values fo irrational numbers never end and do not havea rqpeating pate inthem. ‘i wth itsnever-ending decimal places, isirrational For Example: S75, 2+2, Even numbers: An even number is one thatcan be divides evenly by tivo leaving no remainder, such as 2,4, 6, and 8, ‘Odd numbers: An ccd number is one that does not divide evenly bytovo, such as 1,3, S, and 7. rime numbers: Aprimenumberisa number which can be divided nly by 1 and itself! The prime number has only two factors, | and itself For example: 7,11, 13,17, .. are prime numbers ‘Camposite Number: A Composite Number isa number which can be divided into a number of factors other than | and itself. Any composite number has additional factors than 1 and itself For examples, 6,89 10 Co-primes or Relatively pri having any common factors other than 1 or atest common fietor is Lor 1) ye numbers: A pair ofnumbers not 1. (Oralternatively their For Example: 15 and 28 are co-prime, because the factors of 1S (1,3,5:15), and the factors of 28 (1.2,4,7,14,28)are notin common (except for 1), Twin Primes: A pair ofprime numbers that differ by 2 (successive odd numbers that are both Prime numbers). For Esamplet 83)(5,74(11,13), Place value : Place value is positional system of notation in which the position ofa number with respect toa point determines, its value, In the decimal system, the value ofthe digits is based fon the number ten, Each position in a decimal number has a valuethat is a power of 10, A decimal point separates the non-negative powers of 10, (10/1, (10)!=10,(10}>100, (10)5=1000, ete) onthe lef fromthe ot geet L negative pomersor 10, (10 o2= oys= * 110, HO = F999 ° = To00 efe.)on theright. Face value : The face value ofa number isthe value ofthe number ‘without regard to where it isin another number, S04 7 always has, «face value of 7. However the place value includes the position ofthe number in another number. So in the number 4,732, the hhasa place value of 700, but has a face value of just 7. LIED | : Placeand face valuesorthe digits in the number 495,784: Number [Digit | Place value | Face value 495,7aa_| 4 | 00000 4 3 90000 9 5 5000) 5 7 700) z é 80 3 4 4 a A fraction isknown as @ rational mumberand written in the form of © where p andy are integers and gO. The lower number "g!isknown as denominator and the upper number is known asumerator AD ‘TYPE OF FRACTION! The faction in which numerator isless than the Proper Frac denominator iscalled a proper fraction. 510 ‘or Example For Examples yey) ele . The fraction in which numerator is Improper fraci than the denominator is called improper fraction. 68 3 For Example s 5-21.20 Mixed fraction : Mixed fract ‘wholenumber. onisacompositeots fractionanda higher power, is called a decimal fraction noe 10°10 "i000 ued Fraction: Fractions which contain addition or For Example: c subtraction of fractions or a series of fractions generally in denominator (sometimes in numerator also) are called continued fractions, Formulae to Remember none) 2 4 Sumof first» natural numbers = * 4 Sum of first odd numbers=n? Sum of first even numbers =n(n-+1) Composite Numbers :Itis a natural number that has atleast one divisor different from unity and ise Every composite number can be factorised into its prime factors, For Example :24~ 2 2.» 23, Hence, 24s a composite number, The smallest composite number is 4 Whole Numbers : The natural numbers along with zero (0), form the system of whole numbers, This denoted by W There is no largest whole number and The smallest whole mumber is 0 The number line : The number line isa straight line between nfinity on the left to positive infinity on the right. Number System, HCFand LOM (EEL) 2 : Find three rational numbers between 3 and 5. =4 345 8 Sol, 1st rational number = 2nd rational number (ic, between 3 and 4) 3447 3rdrational number (ic. between 4 and 5) 445_9 Both rational and irrational numbers can be represented in number line, Thus real numbers is the set of the union of rational and ie R=QUQ' Every real number is either mtional or irrational ional numbers, E ‘alent fractions/Equal fractions: Fractions with same Forexample ‘Simple fraction : Numeratorand denominator arc integers, 3.02 Zant 2 as For example Vulgar fact + Denominators are not the power of 10. 395 T2198 ‘example GmZg> Write 2.73 asa fraction a decimal faction, EEE 5: After doing 3/5 of the Biology homewerk on Monday night, Sanjay did 1/3 of the remaining homework on Tuesday night. What fraction ofthe original homework would Sanjay have to do.on Wednesday night to complete the Bio assignment ? (@ Wis ©) (©) 41s @ Number System, HCF and LCM Remaining homework on Monday night ol (0) EXAMPLE (a) Weite21.3751 upto two places of decimal (b) Weite3.27645 upto three places of decimal. is Sol. fa) 213751=2138 (b) 3.27645=3276 1. The decimal expansion of a rational number is either terminating or non-terminating recurring. More over , a al expansion is terminating oF non- terminating recurring isrational, number whose deci v The decimal expansion of an irrational number is non- terminating non recurring. Morcover, a number whose decimal expansion is non-temminating non recurring is inrational, For ecample: ff = 141421356237300508880. 897932 384626433, 14159265: waren smsiatiaitaees Lg 3 Em> O142857 and 2 = 0.385714 Sol. We find by dividing, 4 a by 2: Anumber is divisible by? ifits unit's digits ty by 3: A number is divisibleby3 ifthe sum afitscligits are divisible by 3 Divisibility by 4: Anumberis divisible by4 ifthe last2 digits are divisible by 4 ori the last two digits are O's Divisibiity by A number is divisible by 5 i's unit's digit is Sor0, Divisiblity by 6: A number is divisibleby 6if itis simultaneously dlivisible by 2 and 3 Divisiblity by7 : A number is divisibleby 7ifunit’s place digits ‘multiplied by 2 and subtracted from the remaining digits andthe ‘number abtained is divisible by 7. Divisibilty by 8 A number is divisibleby® ifthe last 3 digits of tho number are divisible by 8, ofthe last throo digits of « number as Divisibility by 9: Anumber is divisible by 9 ifthesum ofits digits is divisible by9. gy by 10: A numbers divisible by 10iFitsunit’s digit is, ity by 11 : Anumberisdivisibleby 11 ifthe sum of digits atodd and even places byl Divisibility by 12 : A number is divisible by 12 ifthe number is, divisible by both 4 and 3 Divisibilty by 13 : A number is divisible by 13 if'ts unit's place digit is multiplied by 4 and add to the: ‘number obtained is divisible by 13 Divisibility by 14 : A number is divisible by 14 ifthe number is divisible by both 2 and 7 fe equal or differ by a number divisible wining digitsand the Divisibility by 15: A number is divisible by 15 ifthe number is, divisible by both 3 and ELGG 8 : 18473512 divisleby 7? Sol. 47331 -2x2=47327 4732-27 =4718 4T1-2%8=455 4s ‘Sis divisible by 7, therefore, 473312 is divisible by 7. (EET 9 : What is the value of M and N respectively if M39048458N is divisible by 8 and I 1, where Mand Nare single sigit integers? 35 @ 74 ) 86 © 64 @ 32 Sol. (¢) Anumber is divisible by8 ifthe number formed by the last throe digits is divisible by 8 i.e. S8Nis divisible by’, Clearly. N=4 Again, a number is divisible by 11 if the difference between the sum of digits at even places and sum of clgits atthe oid places is either 0 or isdlvisibe by I ic. (M494444+8)-(G4048454N) M+ 5—(16+N) = M—N+9 must be zero ort must be divisible by 11 ie. MN. > M=2+4=6 Hence, M=6,N =4 DIVISION ALGORITHM: Divielend = (Divisor= Quotient) + Remainder where, Dividend = The number which is being divided Divisor (Quotient = Greatest possible integer as a result of division Remainder= Restart of dividend which cannot be futher divided by the divisor The number which performs the division proosss a FETLIZISP 10 : A certain number when divided by 899 leaves the remainder 63. Find theremsinder when the same number is divided by 29. @ 5 ) 4 om (€) Cannotbe determined Sol, (a) Number=809Q +63, where Qis quotient =31 «29 Q* ($8+5)=29[31Q+2] +S Remainder Important Al formul. (a+bj=22+20b+b 1 2 (a-b)?=2? Jab +b? 3. (ab) (a+ b)=22-b? 4 at bP +a bP = 2002 + by 5 (a+ b—(a-by?=dab 6 (a+by'=a? +b +3ab(a+b) 34302 + Jab? +b 1. (a-by'=a5— 3a" + 3 ab?—b* a’—b?—3ab(a—b) & +B = (a+b) (a2—ab+b%) 9. B-b= (bya +ab+b?) 10 +b +e} abe = (a +b+ o)(e ab-be—ea) 11, a+b) +e3 abe, ifa +b +e=0 Cyelicity ofa number is used mainly forthe calculation of unit digits 1. Cyelieity of In 1, unit digit will always be 2 Cyelicityor I Number System, HCFand LCM 2! Beas 2-16 264 27=128 28256 After every fourth interval 2,4, 8, 6 are repeated So cycle of? 52,48, 6 BEING 11: Find univ git of 2 Sol, Hire unitdigt will repeat as 2, 4,8, 6 after every four interval tll 2 next $3 willbe2and$4 willbe, Sounitdigit of25'= 4 EIMITGp 12: Find unit digitor2525 Sol. Here, 2,4, 8,6 will epeat afer every four interval till 20 next sit wi be2, 4, [] ETUIZGD 13 : Find nit digit of 331. Sol. Cyecleof3 is 3,9,7, 1 which ropentsafter every fourth intrval will 1331, sonext unit digit wll b3. ELE 14 : Find unit cigit oF 963% x73 Sol. Unit digit of 963°=7 will be Unit digit of 7373-3 So unit digit of 9636 « 7373=7 + 3=21 ied EXT 15 : Find unitdigit ofa" » 6° Sol. Unit digit of4°=4 Unitaigitof6* 6 Sounitdigitof4°~ 68 4x6-2ie4 Number System, HCF and LCM as ER QERCISE |. Theeifference between a two-digit numberand the number obtained by interchanging the digits of the number is 18 The sum of the digits of the number is 12. What is the product ofthe digits of the two digits number ? @ 8 ) 7 © (@ Cannotbe determined 2 There are 1S dozen candles in a box. IFthere are 39 such boxes. How many candlesare there in all the boxes together? (@ 720 (by 660 (© 682 (@ 3510 3. The product of two consecutive odd numbers is 19043. Which is the smaller number? @ 7 () 131 © & (@ 29 4, 2 ofa number is 250 more than 40% of the same number What is the number? (@ 1250 (bh) 118 (9) 00 (@ 20 5 125 6 7 5. tPthe factions 5. G+ py. andy ave arranged in ascending order ofthcir values, which one will be the fourth? 3 » 2 @ oa © > o> 6 How many numbers, besveen 1 and 300 are divisibleby 3 and S together? @ 16 ©) 8 ( % 0 7. What isthe remainder when 4% is divided by 62 @) 0 ) 2 3 @ 4 & —Thonumber 31131131131 31131 311i (a) divisible by 3 but not by U1 (b) divisible by 11 but not by3 (c) divisible by both 3 and 11 (€) neither divisible by 3 nor by 11 9. The divisor is 25 times the quotient and 5 times the remainder. Ifthe quotient is 16, the dividend is 6400) () 68k fe) 400 @ 40 ———— m0. Fre is equal w © 2 © @ © 16, wae nthe 20hours, thenat what time will theyagain changesimultanccusly ? (@) $:20:08hrs (b) 8:24; LOhrs (©) 8:27:12hrs (@) 8:30: 1Shrs a6 2. m6, x. What least number must be subtracted from 1936 so that the remainder when divided by 9, 10,15 will leave in each casethe same remainder 7? f) 9 o » © 9 (ome Find the greatestrnumber that will divide $5, 127and 175 so asto leave the same remainder in each case @ % () © 3 @ » Thence of 28,4 ana 2 pCR Ze and 3 rmuemor 2332 ‘ _ Tes Three numbers arein theratio | Thenumbers are f@) 4.812 ©) (©) 10,2030 @ The prexluct of2 number is 3024 and their LOM is 36 55 2:3 and their H.C. is 12 5,10,15 12,2436 Find their HCF f@ al 8% (©) % (@) None of these (a+b+0)—(a-b-eF = f@) 4ab+e) (©) 3ab+0) (b) 2atb~e) (&) datb—c) 1 le 1) is equal to 1 @) x (@ x Ifx+ + =2andx isteal,then thevalueofi!”+

oe k ot Thepice val fn. 01S935 ol ( t0 ©) vw) tie0 ‘m'and'n'aretwopositiveintegers. fm" =25, then whats the value of n™ ? @ 4 © 0 os @ 2 Sum of number of two digits and the number obtained by reversing the digits of the first number is 110. If the difference ofthe digits is 4, thon the number is @e@ © 3 o 4 @ 31 Diviele 50 by halfancl ade 20, From the same, subtract 35, What do you get? @ 0 © 85 15 (@)_ None of these Number System, HCF and LCM av B, Sum of place values of 6 in 6260618 a_b_e_Ta-db+3e (a) 6066 @) 8 ”. Phen the valueofPis fe) 60608 (6668 1 @ 3 © % 44 How many 5 arein 3? @ 2 @ ® @ 2 6 a1 ate ae Ry 3. =F sthen SP is equal to 45, The sam ofall the fictors of 100 is fa) 2s @ a wt (© 26 om 2 46, Thenumber offictors of 18s ie @ 4 5 @2 @ {c) 6 @ 7 . When 71777 isclivid by 7, the remainders 51, ln number 97580, when thedigits 7 andl Ss imterchanged @ 1 3 its place, then the difference between the original andthe © 5 6 new numbers 4%, 113468 is to be divisible by, then the value of missing (@ 1900 digit, * should be (b) 1080 @) 6 ws (10s 4 @3 (@ 100 _t@te tots Tete Toes Te 7s Te 2 | @ > le [2 [oe fe Te [a2 Te 3 ,@[>s/@fsalels lols lo 4 @ > [els e]™ Tels To sif@[>slm)s|@lsl[o]s | @ ‘*lo,l[«|m)sel@|s«|m|]«|@ 7) o>’) oe] |@l*|el*|@ s | @ [ss | @ [es [ols [ols To > le [slots Tel» Te [ss Te vw lLol»etlef>slolsl@ols]@ INTS AND SOLUTION (Se Sgt = 1-250 *5=1250 ar ft From equation (i) y=7-2=5 130, Required product™=xy=7% 5~3$ 24 2 (@) Total number ofcandles= 15» 12 39=7020 (On arranging the numerators in ascending erder 3. (a) Outof the siven alternatives, 108, 117, 130,156,192, sarees ane Ascending order ofthe fraction is Requifedsmaller number = 137 AB 6 © 2 @ 8 Ww 2b) oR, » © 1. LCMof Sand $15 304 20numbers Let us divide the different powers of 4 by 6 and find the remainder. So remainder for 4) 4°= 4nd soon, cinder for aay power of 4 will be 4 only Sum of digits Now, (Sum of digits at odd places) (Summ of digits at even places) (19- 16)=3,not divisibleby 11 So, the given number isneither divisible by 3 nor by 1 APA P= 4 a4, 44 Hence rem 35 und soit isnot divisible by 3, Letthedivisorbex According to the question Quotient will be= Remainder Given, quotient =16 So, X=16 x=25% 16, “ Dividend =Divisor * Quotient + remainder = x 16 +1} [Puting he valor) 1625405 = ABSA agg Divsoe= 28 «16400 Remainder= #2 — 80 Dividend {400° 16-+80 6480 Let x= 2+ 2+vie wa2+¥ darex xP-2xtx: A(x-2)=0 26+ D=0 x=2oe-1 16, 19, © @ @ o @ o o ©) » Number System, HCFand LCM We have, 14243 ...¢0 mined) +100 100)-(1 5050, 3775 Here," y 2034304 328 12+17=29 1275: Qx2 100 Upto 100 thereare “ 0 oxkd numbers ‘Sum of first 50 odd numbers = (50)? =2800 4 Upto 174 there are “2 odd numbers ‘Sum offirst $7 odd numbers = (87° =7569 agus (7569-2500) ~ 5069 He ofon pene ambi SoLCM=iI 71117 Thelin mambr™ 244-88 The end nrnher= ESL. M264 pp The traffic lights will again change at thres different road crossings simultaneously afer the LOM of 43, 72 and LOS i.e. after every (432 sec) 7 minutesand 12 seconds, ic. theearliest at 8:27: 12 hours. TheLCMof9, 10.and 15 =90 COndlividing 1936 by90, heremainder = 46 But is also a part of this remainder 46-7=39 Let x be the remainder, then the numbers (55 — x), (127 ~ x) and (175 ~ x) are exactly divisible by the required number. Now, we know that if two numbers are divisible by a certain number, then their difference is also divisible by the number. Hence the numbers(127~ x)~ (55), (175—x)- (027 —x)and (178 —x) -(65~x) or, 72,48and 120 are divisible by the required number, HCF of 48, 72 and 12024, therefore the required number = 24 the require! number Required HCE Number System, HCF and LCM a9 LCMof2.3.49 33. (b) Letno.ofmarbles~x B RoquitedL.CM = 24. (a) Letthe required HCE =x.S0,x @ The numbers are 12, 24 and 36. F : 25. (a) Product of2 numbers = Product of(LCM HCF) 5 = 0222 sooiey 3024 54 => W2=36« HEF HCF= = 84 =0.750 26. (a) (atb+e}"—-(@—b-e)? (atbteta—b-o)(atb Ascending order is a(2b+2 mb) 36, (b) 273045 Lx ox 100 7 @ {x+4] i 37. (a) SumofAP.= + 2at(n-tjd) where, n=no. ofterms a=firstterm d= common difference 374+ (12-1)4] =6 (74+ 11 4) = 6-30) =— 180 oe a oy"(Ry" _ PY R 2 RP © oo1s4 Ly 0* 1 BR (xe i eb Bextih The place value of lis 55 > (IP 0SK-1 40. @ ne 225 144 Here,m=5,n= value of n™= 28 = 32 41, (b)_Letthe two digit number be 10x+y » © Reversing the digit, number become loy+x hh, ssum=10x+ y+ l0y+x . Lo lx=ly=110 2225 sade 7 Sxty=10 30, (6) Largest four digitno. = 9999 ces, ft smallest four digit no, = 1000 Bramaau(1) sum= 9999 + 1000= 10,999 ea (©) Required number =2999 -1000= 1999 32, (©) Converting fractions into decimals. 3 5 2 a. (a) = =0375; 3 =0955; $= 0.666; Hence the number i 43, (6) Place values ofall 6in 5 63606 oe +> 60000 600) 5.2 arein ascending order sore Sum 60000 +600 +6 60606 Alo ‘Number System, HCFand LCM 1 1 48, (a) According to divisibility ruleof9, sum ofall he digits 44. @) Lets’ J arein 5 should be divisible by 9 > 344444648 So, *= 62 214+ 6= 27 is divisible by 9, #, 1 diga4 45. (@) Factorsof100=1,2,4,5,10,20,25, $0,100 ‘Sum of ll the fctors of 100 = 1+244+5+10+20+25+50+ 100 27 Tek 4x dk 3x 7k _ 3k P 3 4, @) 2 |18 Pp 2k ~16k+ 21k Hl P T P 18=2*3%3%1 50, Number of factors =4 OTT T SI. (@)_ Original number =97580 Newnumber=95780 Regd. difference ~ 97580 -95780~ 1800 21 | Remainder Simplification : Algebraic expressions contain alphabetic symbols as well as numbers, The process to find the value of given expression is called simplification Rule of Simpli “V-BODMAS’ Symbol Full nam Sign of the symbol V Viniculamn (Ba) B Bracket OU ° of (asmultiply) D Division * M Muhiplication A ‘Addition 8 Subiraction Before applying the BODMAS "rule, we simplify the expression under the viniculam. After removing the brackets, we must the ‘operations in the given order. FETTIEp | : What value should come in place othe question ‘mark (?) inthe following question ? 432466 -15@2~? @ (by 2360 © (a) 26950 Sol. (@) ?=432 « 66~ 1562=28512~ 1962=26950 FETUIEp 2 : What value should come ‘mark (2) inthe following question ? plaeeofthe question 1 2 a) It yt @ i ly 12 s y 12 a 12 o hy w@ 'y ee eres So 72 DS = Sx a ale Simplification EEL 3: What vate shouldcomein placeofthe question mark (2) in the following question ? or 680 =? 1 (o) 10 @ 210 (@) 240 hy 7 = 1680% 180 Gp +: . 6 ws ow % 1 5 2 ot oz @t © 2 3 4 0.73 Sol. @) Yloxe7 06st (a+b) (ab) (0654) ° 0.654 a 6 9, 10. 999.99 +99.99+9.9=? fa) 110397 (by 101989 (9) tin (0) tops To 3715008 =31878 @ 8 2 ©, 6 oe (OP -(12)°=976 @ Ss ® % @ os sss 6205~2 in » 1 2 a $9475 ff =195%5 (a) 360 (si 9 s (3 fi +29= Se a3 5 4 a1 TOT TIAT +077 707474 77=2 (a) M7 (by M668 (3) seas (a) 94686 The value of 3+ * is i 0 2 » % » 2 w * w 2 Ea y 4 o * o 4 The value of(243)% (243) sequal to fa) 016 3 1 ot (@ 08 1F*381 is divisible by 11, then th tat the place of is, f@ 0 ® 1 4 @ 7 The sum of first 20 odd natural numbers is equal to fa) 210 (b) 300 (©) 400 @ 2 17172 isdivided by 18, theremainder is fa) 1 ) 16 1 @ 2 1 Given that 0.111 + 0.44 isequal to 5 m L @ 3 oF l- 4 16. Simplification ERCISE Twonumbersare inthe ratio 17: 45, One-third of thesmaller 1 is lessthan — ofthe bigger by 15. The smaller number is 1 od f@ 255 ) 675 © (a) 865 1 Twonumbersare respectively 12=% and 25% more than a third number. The first number as percentage of second ‘number is @ 0 © @ 75 @ 9 Three numbers arc inthe rato of 3:2: Sandthesumoftheir ares is 1862, The smallest of these numbers is @ 4 ) 2 ou (@ % A number consists of two digits such that the digit in the ten’s place is less by 2 than the digit in the unit’s place 6 Thros times the number added to times the number obtained by reversing the digits equals 108. The sum of digits in the number is @ § 9 © 6 @ If* moans adding 6 times the second number tothe first ‘number then (1 *2) "3 equals @ 2 (h) 3 ) B @ 9 ‘998 The value of 999275 999 is ‘999 fa) 990809 (b) 998996 fo) 999824 (@) 998999 4949 49% 49 =7) @ 4 ® © 8 @ 16 16.02 <0.001=? (a) lem () ooo (9 L021 (oom 260s © @ ) 2m (@ 2» &) 36 @ 2 Simplification as 2S Whot should come in place of question mark (2) in the @o Ht fllowingequation? 1 1 Sy @5 @5 n 9 2 ® my 28 is equal to: 9 @ 1017 1 u ® 3s faroonor225 . 26 is equal to 1 1 0.0000: ‘ @ 45 @ st ) ‘ » o = @) 70 35 © 07 @ 24 30. (800+ 64)x(1296 +36) =? lay (a) 420 b) 40 9 00 (eo 40 tt@{«fol[ulflof«lofsl@[ x» Lo 2/@|7/o;efoel|"lel2 lol” lw 3 b@ ts Po fs P@ [is Po [2s Po Ts Po

no. oftimes P repeated) NOTE: If factors of P aren & (n+ 1) type then value of fp Pedro (n+) AG Surds, Indices, Square Roots and Cube Roots ER QERCISE 1181-41520 (267 +¢ @ 8 on 4 @ o» o7 @ [i089 RD ~ Vf @ @® (b) 2500 © (@ 1378 (= 132)+ @ 9 3 @7 (6 19)! 98 «(499 =(19y @ 4 (8 © 16 @ 2 Tix +y=28andxy= 126; then (x)? + (9)2=? @ i) o {@ Cannot determined @ ( © (@) 207% 19(26)? is subtracted from square of anumber, the answer so obtained is $49, What is the number? @ 3 (b) 33 © @ 41 143 yy+aP=2 (a) 2005 (b) 39005 © (a) 29.082 @ % (by 25 © @ 15 a FF 1 8 @ id 1b 1 oy @» 124x128 =2 @ 2 (b) 12 @ 12" (@ 127 19150 19!46=9 f) 10° (b) 100000 (© 100 (10000 16, 1, 2, What should come in place ofthe question mark (2) in the following equation? (x9 49 @9 () 2 © 3 @ 4 What should come in the place of the question mark(?) in the following equation? ver 48x BS @ 8 1 (© 28 @9 ‘What should come in place ofthe question mark (?) in the following equation? a7 a2 car? =) 1 @3 24 6 @3 Evalute: OF —VF @ 4 ® © 16 @ What could be the possible one's digits ofthe square root ofthe number 9801 ? fa) 9 39 33 oo ‘sal Thesimples fam of is @ os 008 (©) ws (500 fir aise aS =2 @ 3 5 © 8 om pare irae + Jaa025 =» @ 2 2 © 1 (@)_ None of hese Find the greatest number of ive dis whi i pert (a) 5683 tb) so (©) 9856 Ce) 9865 If JS" — 729, then the value of nis @ 6 8 © 8 @ 2 Surds, Indices, Square Roots and Cube Roots aay 4 14 then x sequal to Its 3+ 22, thon thevalueof ( =) Ww o B @ ® (@) None of these © @ 5 1 1 1 l on & Wo Ww i By USE tina es le 4 @ 6 5 Bewal to wad wo ® @ 0 wa 0 5 3 \ @ a © @ + 26, By wha! en number 675 by lied w obtain number 2 2 which isa perfect cube ? @ 5 © 6 Fa ? 7 @8 2. Yonoa96 @2 @ 4 04 : (9, 008 000 2 2124-14281! = 198, then find the value ofx ©. OF, oe 36. Ifa?=64, then find (aby * + pe @ 4 h 2 ». 22 and z= x*, then find the value of abe, ©! @ 0 (b) 0 fa®ane)” #1} oe 37, Ufa=2,b=3,thon (a+b) is 30. Simpy i @ 4 3 1 © @ 3.194871, then valucof is @t 0 etl py © 4 @ -3 a. (2) -(2) 9. Find repro o #125 @ 1 @ ys 98 7 © 9 @ 2s 2 @ | [wm | 2 | @ [39 [i @ [20 Fe [30 Te als Surds, Indices, Square Roots and Cube Roots INTS AND SOLUTIONS L @ © ” 4 5 6 & 2 @ & @ % mo @ 1 @ © (P= 6? = 1181 +1520 =P 4676-2701 = P=2701 676 =2025 => Sms =45 (I) (3439928 (714 i089 + 289 15 2=(50)?=2500 Pac 13)= =(14+13)(14-13)=3 ay or (isy'= Soe ar (19)? =(19)24 = (1916 ar?=i6 xt y=23and xy= 126 Now x2+y7=(x+yP (237-2 x 121 529-282 f-8-8 = Peele es= a4 (144 20736 According o th 2549 he question, 2 -676= 549 ee P5494 6 «x= JS 35 2= (4) x Sy +45 #x5'_ st , SxS x5 ad 1035 “Ye? 39,0625 103s 45 (12+4)(12-4) _ 168 Or 3K9 2= 124% 1213 = 12 9 16 9 24, @ 105= 10! =7 19% Ce 10*= 10000 85645427827 (a 2= SxAS2IAT gy © a =(4n (a) V64—-Yod > 8-4-4 = —————— 0) irs Jains vie = ir Greet = Virion =5 © fini 2 eS =VE SP = 5x31 158 Hence, ¢= JIS4=155 = v9 = ITT =17 (©) Greatest number ofS digits is 99999, 99 (316 Required number=(99999~ 143 = 99856). w coe x _ 196 © * 169 169 “eo ** (Bas) Bi) (BF © Gvenenp. EAH (Weve) * (Re) Wey Surds, Indices, Square Roots and Cube Roots aag 1 (Serv) 1 1 ay vw (ve-+) -xt2 EB 8) TEA (Ra) os 7) (v7 +v6) 5 + i) “Gost al Ls ea { (5-28) * 5-22) = [oe -(R dF} (FF 8] 2-4 +( V5 eV) = (9 4NF) 342 26. (a) 6752545034343 Tomiakeita perfct cue, it must be multiplisd byS ~—_ Bn @ eyes Ald 2. (b) Joona = fo1e =04 HOY =243=38 xtyaF uli) 2% (bh Bug! (On solving @) and (i), weget x4 MQ) Leal 36-4 kes2= kx3=4) and6=k 7 Now 2x3=6 <> k¥ xk! =k # ook %. (b) Given Expression giro etent 8. @ Using eden <2 g(a bya2 47a) (b-oyb2e2sbe) %. tio is strictly a mathematical term to compare two similar jantities expressed in the same units The ratioof two terms *x" and ‘y'isdenoted by x: y In general, theratio ofa number x toa number yis defined asthe quotient ofthe numbers x and: The numerator ofthe ratio is called the antecedent (x) and the denominator is called the consequent (y) of the ratio. EXAMPLE] Which of the ratios 2:3 and S : 9is greater? Sol, In the form of fractions the given ratios are 5 and 5 3 Reducing them to fractions with a common denominator 6 theyare written as 2 and > 9 6 Hence the greater ratio is 2 or 2:3. Compound ratio: Rutiosarecompounced by multiplying tagether the antecedents for a new antecedent ancl the consequents for new consequent. The compound ofa: bande: dis #8 ,ie., a0: bd. bxd FETUIZG 2: Find the compound ratio of the four ratios 4:5, 15: 13,26: 3and6: 17 wisi Prog ey kd bad ac “ EE epee ercay a atectere ea a aes Dire ene! er iheraio ofan a eansequents of two equal ratios are equal, This property iscalled Alternendo, ¢ avb_etd 3 1f 222, then S624. this property is pea Ob a ee called Camponendé. Ratio, Proportion and Partnership a ab 4 if ae This property is . e Property called Dividendo, ale atb_e+d 5 if 2°, then 22%, This property is We BaS, thon S22 =S5 This propert called Componende—Dividendo. 6 £. ‘Then, f sum of Numerators Each ratio= Sm of Denominators bad SbeaEtES GEEMEP s:$ soso acer 2 tay os B GNGP 4: Find the valueot 24 248 its ab x __2b Sol xr eb a atb By componendo ~ dividend, a ab GIMP §: The ratio of the radius of two circles is 2:5 Find the ratio of their areas, Sol. Ralioof their areas 4:25 Ratio, Proportion and Partnership When two ratios are qual, the four qu are said to bein proportion, We b & sian esdaciamnitins Remember Er gett ie pr Ue eto ct cxicoe w euale yee sien eee ie ba Toveetaniensberiewetarcocund porate nearer ac= OF Pacis estonia ites unity we prpoel ten s ote = ad=be. third is the duplicate ratio ofthe first is to the second. IP azbe:b:ethena:c—a?: ntities composing them aa GZ + Sol. Let be thorequired mean proportional. Then, Bixuxs 5 x J5n7S =15 To find the values of an unknown when four numbers are in proportion, EXAMPLE numbers 10, 18 Find the mean proportional between 3 and 75, What must be added to cach of the four 38 so that they become in proportion ? Sol. Let the number to be added to each of the four numbers box Bythe given concition, we get (10 #39: (18-43): 222+): 88H) > (O+)G8+N)= (I8+2) +N) > 380--48x+x2=396-+40K-Hx2 Cancelling x? from both sides, we 380+ 48x=396 +40, > ABe- 40x 396-380 16 2 should be added to each of the four given = B16 x 2 Therefore, umber, Anz 1 10, Ratio, Proportion and Partnership ERQERCISE 190.75: x::5:8,then is equal to @ 12 (b 120 125 @ 0 There are 240 doctors and nurses at hospital, M'the ratio ofidoctors to nurses is 5:7, then the nurses atthe hospital 2» &) © (~ m @ A bag contains ® 216 in the form of one rupee, 50 paise and 2Spaise ecins in theratioof 2:3 4, Thenumber af 50 paise @ % Ms @ us @ mi WA: B:C=2:3:4, th ALB.C 1 rf 23:4, thn 2:2 isequalo fa) 4:9:16 ( 8:39:12 (©) 8:9:16 (Wo) 8:9:24 A sum oftmoneyis to belistributed among A, B,C, Din the proportion of5 :2:4:3. 1fC gets® 100%) more than D, what is B's share? (a) %500 (b) T1500 (©) %2000 (a) None ofthese A began a business with € 4500 and was joined afterwards by Beith ® 5400. Irtheprafitsat theend of year wasdivided in theratio2 =| then B joined the businessafter fa) Smonth (b) 4months (c) 6 manths (@) Tmonths on dividing €581 among A, Band C such that fur times N's shhare is equal to 5 times B’s share which is equal fo seven times C's share. Then shareof Bis: a) ms () 196 ©) 0 @ 4 112: 18 s::24, then find the value of f@) 8 (b) 16 ) 2 4 The ages of Aarzoo and) Amay are in the ratio of 1:13) respectively. Afler 7 years the ratio of their ages will be 20:23, What is the difference in years between their ages? (a) years () 7 (©) 6 years (OS years The alioofthe radius of wo circlesis? : 5, Find the ratioof their areas. fa) 2:5 (b) 8:20 (©) 4:20 (@ 4:25 The sum of Wo numbers is 60 and their difference is 6 What is the ratio of the two numbers ? f@) 1:10 (b) 9:11 (o) 11:9 (@ 10:1 One litte of water was mixed to 3 litres of sug of sugar. What isthe peroentageof sugar in containing 4# the solution? 3 () 4 6 (@)_ Insufficient data 16, 2, and B enter into partnership with capitals in theratio3 = 4, At the end of 10 months A withdraws, and the profits now aredivided in the ratio ofS: 6, Findhow long B remained in the business? (a) O months , (6) 6 months @ When 50% of ane number is added toa second number, the sevond number increases to its four-thirds. What is the ratio between the first number and the second number? f) 3:2 ) 3:4 () 2:3 (@) Datainadequate Ifa dividend of & $7,834 is to be divided among Meena, Urmilaand Vaishali in the propertion of3:2:1, find Urmila's share, @) t19281 () t17380 (©) 223,133 @ Z9278 A and B started a business by investing & 35,000 and % 20,000 respoctively. B left the business aficr $ months and C joined the business with a sum of € 15,000. The profit cared at the end of the year is € $4,125. What is B's share S months of profit? f@) 214133 ) %s,00 (©) 213,460 (@)_ Cannot bedetenmined MA:B=3:4,B:C=8: WnadC:D=15: 17 Then find: B:C:D. @) 921213211 (b) 4:5:6 (oe) 9:12: 18:17 (@)_ None of these And B renta pasture for 10 months. Aputs in 100 cous for months, How many cows can B put in for the remaining 2 months, if he pays half'as much as A? @ ®) «0 (© = 1000 There are two numbers sich that the sum of twice the frst ‘number and thrice thesecond number is 141 and the sum of thrice the first number and twice the second number is 174, Which is the larger number? ® 2 ) 6 oO #® Ww 4 Populations of two villages X and Y arein the ratio of 5:7 respectively. If the population of village Y increases by 5000 and the population efvillage X remains unchanged the respective ratio oftheir populations bacemes 25 : 36. What is the population of village X? (a) 625000 (b) 675000 (©) 875000 (@) 9.00000 is equal to: 2 Ratio, Proportion and Partnership The ratio of third proportional 10 1 proportional beiwoen 9 and 25 is f@) 2:1 (b) 5:1 () 715 (a) 9:14 A sum of money is tobe distributed among A,B,C, D inthe proportion of :2:4:3. FC gets Rs 1000 more than D, what and 30 and the mean is B's share? fa) Rs500 (b) Rs1500 (e) 52000 (None of these 24 930 is divided among A, B and Cin such a way that their Laid shares may bein theratio 5° * What share did A get? 24's f@) 40 () 400 (©) 09 @) 60 25, If2A=SB=6C, then A: Cisequalto fa) 4:35 (b) 3:1 (o) 3:2 (4) Cannotbe determined 26 Find thetriplicateratio of 2x: 3y f@) xy (b) 88:27 (©) 42 (4) Cannot be determined 27. Find the value of k that mustbe added to 6, 13, 30and 58, 80 that they aren propertion is fa) 2 () 3 (©) 6 (4) Cannotbe determined 28 Ina class ofS students the ratio of boys and girls is? : 3, How many more boys areto be added to make the ratio 1:1? @ 6 ) 2 fo) 15 (@)_ None oF these 29. Find the ratio ofthe perimeter ofthe trang the each side of triangle and square is equal. f@) tr (b) 3:4 fo) 1:2 (4) Cannot be determined A sum of money is to be divided among Z, X, ¥ in the respective proportion of 4:5:6 and another sum tobe divided between Aandl B equally. IZ. gor® 2000 less than A, how anda square if much did X got? fa) & 10,000 () 5,000 (©) & 4.000 (a) Cannot be determined 31 The total number of boys ina school are 16% more than the total number of girls inthe school, What is the ratio ofthe total number of boys to the total number of girls in the school ? fa) 2821 (b) 2935 (0) 2529 (4) None of these Ratio of the earning of A and B is 4:7 respectively. Ifthe earnings of A increase by S0% and the earnings of B decrease by 25% the new ratio of their earniny 8:7 respectively. What are A's earnings? (a) €26,000 (b) 828,000 (o) 21,000 (4) Data inadequate The total number of students in a school is 3250. Ifthe numberof giels in the school is 1495, then whats the ratio ofthe total number of boys to the total number of the school? (a) 23:27 (© 27:23 ro} @ 25:29 29:2 * #0, 4 Aa The average age of a woman and her daughter is 42 years The ratio of theirages is 2:1, What isthe: @ () 48 yoars (©) 52 years (@) 32 years aughier's 28 years Ninad, Vikas and Manav cnter into a partnership. Ninad invests some amount at the beginning. Vikss invests double the amount sfter 6 months and Manav invests thrice the ‘amount invested by Ninad afler 8 months, Theycasn a profit, ‘of 245, 000 at the end of the year. What is Manav’s share in the profit? (a) % 25,000 () % 15,000 (© % 12,000 (@ %9 p00 In.a mixture of milk and water the proportion of water by ‘weightwas 75%, Ifin the 60 gms ofthismixture 15 gm. water ‘was added, what would be the percentage of water? (15% () 80% © % (@ 100% Sarita started a boutique investing an amount of € $0,000, Six months later Neeta joined her with an amount of = 80,000, yetr they earned a profit of 18, 000, What is Sarita's share (@ = 9000 () © 8,000 © ©1200 (@ @ 10000 The ratio beween the boys and girls in a class is 6 : 5, 1F8 At the end of on theprofit? more boys join the class and two girls leave the class then the ratio bocomes 1: 7. What is the number of boys in the class now? @ 3B © 38 o4 @ 36 Theratio ofthe ages ofa father and son is 17:7, 6 yearsago the ratio oftheir ages was 3: |, What isthe father's present () st (4) Cannetbe determined gest and the second largest angles of a tia inthe ratio of 13: 12. The smallest angle is20% of the sum of the la sum of the smallest and the second largest angles @ (o) 108 (© 100° (@) 1 Mr. Pandit owned 950 gold coins all of which he distributed amongst is three daughters Lalita, Amitaand Neeta Lalita ave 25 gold coins to her husband, Amita danated 15 gold The new respective ratio ofthe coins let with them was, est and the sevond largest angles, What is the nd Neeta made jewellery out of 30 gold coins, 20: 73:83. How many gold coins did Amita receive from Mr Pandit? @ 0 © 0 ®) @ ais 350 Aw Ratio, Proportion and Partnership 1tTotutota tl ofa Te. sa Tae 2 [w]e lw [2 [ow [2 Lo 3.) es f@ > ets Le 4 @ is Te [2 Te as se fs (@ [2s Tm Ts 6 @ [is Te [26 Te 36 7 Lo) Lela) we] 7 | @ s |] is [om [2 7 @ [3s To 7 Lele» To [2 To [3 | 1 iw 2 Tae Pe ae Te To INTS AND SOLUTIONS 6 o @ o @ iC) @ (b) 4 times A's share 6 x= Sa1.20, (0.758) Number of nurses: x 240= 140 2 Letthe no, ofone rupee, $0 paise and 25 paise coinsbe 2x, 3xand 4x respectively “According to question, 216 ‘Number of50 pase oins= 4x 3= 144 LatA~2x,B=3xandC~ 4. Then, A_2n_2 B33 4048 Box a AO ABC 23.2 go. BCA 347 Let the shares of A, B, C and D be ® Sx, 2s, € 4x and 3x respectively. Then, 4x ~3x = 1000 = x =1000 BB's Share =% 2x ~% 2000 Let Bjoined after x months. Then, 4500 » 125400 (12—x)=2:1 4500.12 $400x(12-x) 1 45412 Stimes B's share 7 times C's share A'sshare _ B'sshare_ C'ssh 28 20 (dividing by LCM of 4, Sand 7, ie, 140) AsB:C=35:28:20 10, U1 © © @) ) ) Shareof A 20 share ofB~ 7 «581 -€ 196 : 2 sett share ore = 22 «581 =@ 140 aetsssai2s > [22 18x Lot the present ayes of Aarzoo and Amay be 11x and. 13x yeurs respectively According to the question, Let? _ 20 8 Bxt7 23 260 +140=253x + 161 cr, 260x -253x~ 161-140 Lx 2*3=6 yours Difference between their ages = 13% Ratio oftheir area The requited ratio of thenumbers Soril:9 9 in(3 + 1) lire solution =0.03=3 Initially A's investment = 3x and B's investment =4x Let B remain in the business for ‘a’ months. 3 Axx 104s n= 5:6 3x10 6=4x xn x5 Ratio, Proportion and Partnership “4 16, 19. 20, 2. © © © © ®) o Let the numbers be andl x respectively yar 4 x+5M% of 7= (2esrem)-e1928 m4 Share of Urmilain dividend { ‘ Ratio of equivalent capitals of A, B and for 1 month 35000 « 12: 20000 » $ £15000 7 =35% 12:20 5: 15% 7=84:20:21 ‘Sum ofthe ratios = 84+ 20+ 21 = 125 20 2 Bsshare= { j3-*84125 |=13460 A:B=3:4 B:C=8:10 C:D=18:17 AB:CD=3 BIS: 81S 11S 10517 =9:12:15:17 %, Suppose B puts in x cows, Theratioof A's and B's rents vteterders then, 1 = 100383 - 6o9 co Let te frst number x andthe econ number by. Acterding tote ston % Beeay Hl a 425-1 ‘i Byequaton ()*3~(i)%2, weave 6x89 6x dy=a23- 348 sxyatsssys tins ». Frememution@), 22-43% 15 > 2x= 141 -45=96 => Larger number=48 Let the population ofthe village X be Sx 3 and that of village Y =x According tothe question, =— 1815+ 25 «25000 5x 25000 2 4250) tate atime Let the third proportional to 12 and 30 bea. Then, 12: 302:30:x-¢9 12x= 3030 (2030) 2 © © O} © om) © © © o © ‘Third proportional to 12 and 30 Mean proportional between 9 and 25 = VO=35 Required ratio= 75: 155: | Let the shares of A, B, Cand D be Rs Sx, Rs 2x, Rs 4x and Rs 3x respectively Then, 4x 3x = 1000 x = 1000 BYsShare= Rs 2x ~ R52000 Thema which fo maneya diel Ll ing =fef et ot0:5:4 (uipyingeahrationy20) Aeshare =® (124950) 500 9 Let2A=5B=6C=k Triplicateratioa?x: y=x2:y Triplicate ratioof 2x: 3y (2x): Gy)" 8 (+k): +b) =: BO+K (584K) > (6+k) (58 ~k)= (13+K) 0k) D+ Otk+ M8 =k +43k+300 > 2Uk=42=3 k=2 Thus, 2 must be added. The number of boys 18 boys The number of girls 218i Now, when 9 more boys are added the new ratio brecames=27:27= 11 ‘9 more bays are to be added Lot each side of triangle and each side of a square be xunit. Then, perimeter ofthe triangle perimeter ofthe square =4. Required mtio= 3x 4x 3xand 3:4 Lot thenumbsr of girls x Number of boys = 1.16 Required ratio 116: 100 Accord Li6x:x 29:28 to the question, 0 and = «i, a2 100 From equations (i) and (ii), wecannot find the earning, of Aand B. Total number of students ~ 1495 Ratio, Proportion and Partnership “4 16, 19. 20, 2. © © © © ®) o Let the numbers be andl x respectively yar 4 x+5M% of 7= (2esrem)-e1928 m4 Share of Urmilain dividend { ‘ Ratio of equivalent capitals of A, B and for 1 month 35000 « 12: 20000 » $ £15000 7 =35% 12:20 5: 15% 7=84:20:21 ‘Sum ofthe ratios = 84+ 20+ 21 = 125 20 2 Bsshare= { j3-*84125 |=13460 A:B=3:4 B:C=8:10 C:D=18:17 AB:CD=3 BIS: 81S 11S 10517 =9:12:15:17 %, Suppose B puts in x cows, Theratioof A's and B's rents vteterders then, 1 = 100383 - 6o9 co Let te frst number x andthe econ number by. Acterding tote ston % Beeay Hl a 425-1 ‘i Byequaton ()*3~(i)%2, weave 6x89 6x dy=a23- 348 sxyatsssys tins ». Frememution@), 22-43% 15 > 2x= 141 -45=96 => Larger number=48 Let the population ofthe village X be Sx 3 and that of village Y =x According tothe question, =— 1815+ 25 «25000 5x 25000 2 4250) tate atime Let the third proportional to 12 and 30 bea. Then, 12: 302:30:x-¢9 12x= 3030 (2030) 2 © © O} © om) © © © o © ‘Third proportional to 12 and 30 Mean proportional between 9 and 25 = VO=35 Required ratio= 75: 155: | Let the shares of A, B, Cand D be Rs Sx, Rs 2x, Rs 4x and Rs 3x respectively Then, 4x 3x = 1000 x = 1000 BYsShare= Rs 2x ~ R52000 Thema which fo maneya diel Ll ing =fef et ot0:5:4 (uipyingeahrationy20) Aeshare =® (124950) 500 9 Let2A=5B=6C=k Triplicateratioa?x: y=x2:y Triplicate ratioof 2x: 3y (2x): Gy)" 8 (+k): +b) =: BO+K (584K) > (6+k) (58 ~k)= (13+K) 0k) D+ Otk+ M8 =k +43k+300 > 2Uk=42=3 k=2 Thus, 2 must be added. The number of boys 18 boys The number of girls 218i Now, when 9 more boys are added the new ratio brecames=27:27= 11 ‘9 more bays are to be added Lot each side of triangle and each side of a square be xunit. Then, perimeter ofthe triangle perimeter ofthe square =4. Required mtio= 3x 4x 3xand 3:4 Lot thenumbsr of girls x Number of boys = 1.16 Required ratio 116: 100 Accord Li6x:x 29:28 to the question, 0 and = «i, a2 100 From equations (i) and (ii), wecannot find the earning, of Aand B. Total number of students ~ 1495 £26 Number ofboys 3250-1495 Required ratio= 1755 : 1495 =27 :2 34 (a) Let the age of woman be 2x years and that of her sdaughterbex Acvording to the question, 2etx=2 x2 ee 3x84 84 35. (b) Ratioofprofit= 1 12:26:34 1 Manav's share = 45000% > =8 15000 36. (b)_In60 gms mixtureproportion of water = 0x asym 100 “58 Total proportion of water in new mixture 45+ 15-60 gms, Percentage of water +100 =80% an 3%. (@) Ratio ofcapital 50000 * 12:80000<6=5:4 180005 Sarita's share= “75. =F 10000 Ratio, Proportion and Partnership Let the original number of boys ancl girls be 6x andl Sx respectively 6x8 11 Then, 59 8 = r= Bu6 B Number ofboys 6x64 8=44 Lot the present age of father and son be 17x and! 7x respectively. Then, > 2x-17x=18-6 > x=124=3 Father's present 17 3=S1 years, Smallestangle = (13+ 12) 2" = 5 Smallest angle = (3+12)x 2 = Ratiofangles= 13: 12:5 Sum of smallest and second largest angles 180x(12+5) _ 180417 (B+12+5) 30 Required number of gold coins 102! 1950-25415 +30) «73 (204-7348: +15 =365+15 =380 Inthe puremathematieal sense, an average isa calculated value, obtained by adding values and dividing the sum by the number (of values, In English, this valueis called the mean, Hence, average Sumber oFelements EQUI) 1: To find the average of 3, 5 and?. Sol. Step 1: Find he sum of the numbers. 34547215 Step 2 Thereare 3 numbers ‘aleutatethe total number, Step 3: Finding averua Tear weed Was & Sumof element EX 2: theaver erage * no, of elements of marks oblained by 4 students in ‘class is 65, Find the sum of marks obtained? Sol, Here, number ofstudents=4 Average=65 ‘sam of marks obtained =65 * 4=260 TMT 3: 1 the sum of elements and avern respectively 65 and 13, then find the number of elements, Sumofelements Sol. Number ofelements= Average Average, Problem on Ages EEINID 4: There are 30 students in aclass, Theaverage age ofthe first 10 students is 12.5 years. The average age ofthe next 20 students is 13.1 yoars, Find the average ageot the whole clas. Sol, Total age of 10 students = 12.510 125 years Total age of 20 students = 131 «20=202years Average age of 30 students 387 57-129 years EG 5: Theav ky. if'the weight of the teacher is included, the aver ge Weight of24 students ina class is 35 rises by 400 gis, Find the weight ofthe teacher Sol, Total weight of 24 students (24«35)ke= 80 ke Total weight of 24 students and the teacher (25% 354) ke= 885k Weight ofteacher (885-840) kg= 45k emule mber © Theaverage offirst natural numbers © The average ofthe first n consceutive even numbers =r!) © Theaverage ofthe first consecutive odd numbers = © Theaverage of the squares of the first n natural numbers 1 can be shown tobe § (n+ 1) (2u+1) A28 x 10, Average, Problem on Ages ER QERCISE The average of 5 consecutive odd numbers A, B, C, is 45, What is the proxtuct of Band D ? (a) 2107 (b) 2205 (©) 1935 (@ mi Find the average ofthe following set of scores 566, 455, 231, 678, 989, 342, 715 f@) 50 & (©) 58 (@ 58 Theaverage age of five officers in a department is 32 Ire of their supervisor is added the average is increased by 1. What isthe supervisor’ a Dand (a) 32 years (b) 48 years (©) 38 years (@ 42years [77a +47b= $452 then what is theaverageof a and b? @ u () BS (©) % (8% The average of five numbers ig 281. The average ofthe first two numbers is280.and the average ofthe last twonumbers| is 178.5, What isthe third number? (a) 48 (b) 336 (o) 28 (@ 468 Find the average ofthe following set oF scores, 965, 362, 189,248, 461,825, $04, 734 (a) 476 (by so () 461 (@ 34 Out of three given numbers, the first number is twice the socond and thrice the third. Ifthe average of the three num bers is 154, what is the difference between the First andthe (@) 126 (H) 2 (©) Ks @ 2 Average weight ofthree boys P, Tand Ris $4 kgs while the average weight ofthree boys, T, P and Gis $3 kgs. What is theaverage weight of? T,R, Fand G? (a) Sa8kys (by S24kgs (c) Cannot be determined erage of the follow 221, 231, 441, 359, 665, 525 (a) 39 (b) 48 (©) 407 (@ ais Theaverage ofS consecutive even numbers, B,C,Dand E is $2. Whatis the product of B and E? fa) 216 (b) 2988 (©) 3000 (@ 2800 The average of 5 consecutive numbers A, B,C, D and Eis 48, What is the product of A and E? (a) 2162 (by 2208 (©) 224 (@ 20 16, 19, Out of the three given numbers, the first number is twice the second and thrice the third. Ifthe average of the three numbers is 121, what isthe difference between the firstand the third number? (@ o) 9 on (@ 44 Theaverage weight ofa group of 75 girls was caleulated as, 47 kgs. Itwas later discovered that the weight ofone ofthe girls was rond as 45 kgs., whereas her actual weight was kgs. What is the actual average weight of the group of rls? (Rounded offto to. (@) 46.73kgs () 46.64 kgs (©) 45.96kgs (@) Cannot be determined The sum offivenumbersis 290, Theaverage afthe first to rnumbersis 48,5 and the average of las two number i What isthe third number? @ 2 () 8% ©) % (@ 108 The average of four positive integers is 73.5. The highost integer is 108 and the lowest integer is 29, The difference between the remaining two integers is 15. Which of the following isthe smaller ofthe remaining twointegers ? @ 9 ) %6 on (@ Cannot be determined The average monthly income of a family of four earning, memhers was 15,130. One of thedaughtcrs in the family got married and leit home, o the average monthly i of the family came down to 14,660, What is the monthly income of the martied daughter? (@) #15350 () £12,000 (0) 316,540 (@ Cannot be determined A seqquence of seven consecutive int average ofthe firs five given integers isn, Find th of all the seven integers @a ven. The atl 1 o = The average of 11 rosults is $0, If the average of first six result is 49 and that of last six results is $2, find the sixth oa result @ 2 wo ©) % @ 6 ove, (x + 2) trees yield 60 nus per year, x trees yield 120 nuns per yearand (x 2) trees yield 180 nuts per year If the average yield per year per tree be 100, then x o 4 @ 6 Average, Problem on Ages 2. 30 pens and 75 pencils were purchased for Rs 510. Ifthe average price ofa pencil was Rs 2.00, find the average price of pen. (a) 210 tu (©) @12 (@) cannot be determined The average of 20 numbers is zero, Ofthem, at the most, how many may be greater than zero? f@ 0 1 (©) 10 @ 9 Theaverage ageofA and B is20 years. FC were toreplace A, the average would be 19 and if C were to replace B, the verge would be 21. What are the age of A, Band C? (a) 22,18,20 (by 20,20,18 (©) 18,22,20 (@) None of these The average age ofa group of person going for pieni is 16 years. Twenty newpersons with an average age of 1S years join the group on the spot duc to which their average becomes 15.5 years. Find the number of persons initially oing for picnic a) D (18 () 2 (None of these The average age of 36 students in 9 When teacher's ageis included toit, the average increases. oup is 14 years, by one. What isthe teacher's age in years? f@ 3 (b) 36 © 3 (@) cannot be determined A batsman in his 12th innings makes a seore of 65 and thereby increases hisaverage by 2runs, What ishis average afier the 12th innings ithe had never been ‘not out"? @) 2 () (o) 4 4s The average number of printing error per page ina book of 12 pages is 4, Ifthe total number of printing error in the first 302 pages is 1,208, the average numberof printing crrors per page in the remaining pages is (a) 0 (4 (©) 810 (a 9 The average of a batsman for 40 innings is $0 runs. His highest score exocedshis lowest score by runs, these two innings are excluded, hisaverage drops by 2 runs, Find his highest score fa) 1 1B (0) 174 (@ 175 A batsman makes a score of 87 sunsin the 17th inning and thusinereases his average by, Find is average after 17h inning. fa) 36 (b 39 () 2 (@ 45 Nine men went oa hotel, Sof them spent® 3 each over their meals and the ninth spent Rs 2 more than the aver expenditure of ll the nine, The total money spent by all of them was (a) £26 (b) 40 (©) £29.25 (@ 27 The mean of 30 values was 150, It was detected on rochecking that one value 165 was wrongly copied as 135 for the computation ofthe mean, Find the correct mean, f@ 151 (by 149 (©) 182 (O None of these a x. 0, A290 Acar owner buys petrol at Rs 7.50, Rs 8.00 and Rs 8.50 per lite for three successive years. What approximately is his average cost per litre of petrol if he spends Rs 4000 each year? (a ts wo zo (6) F798 (a &850 A family consists of grandparents, parents and three grandchildren, The average age ofthe grandparents is 67 years, that of the parents is 35 years and that of the of the tgrandchildren is 6 years. What is the average a family? years (o) 312 years years (@) None of these oftwonumbersis XY. [fone number is X, then the other number is Y @y o> (© 2xY-x @ xw-1) The average of 1Onumbersis 40.2, Later itis found that two numbers have been wrongly copied. The first is 18 greater than the actual number and the sevond number added is 13 instead of 31. Find the correct average (@) 402 () 404 (©) 406 (@ 408 Ifa, b, od, ¢ are five consecutive odd numbers, then the averagein termsofa’ will be @ a2 () at3 (© a+4 @a [the mean soore in mathematics ofa class of $0 studonts is 45, then the total marks obtained by them are @s ) 95 (©) 20 @ 9 The: age of 8 boysina class is 15, Theaverage off group of 15 boys in the class is 16 and the average a of another 25 boys in the class is 14. What is the avera age of the remaining boys in the class ? @ 1525 4 (© 14.75 (@) Cannet be determined The average of four consscutive edd numbers is 36, What is the smallest of these numbers ? @ 31 () 35 Os (@) None of these Thetoual oftheag ses ofa cassof sirlsis 1050, theaverag age of 25 ofthem is 12 yrs and that of another 25 is 16 yr Find the aver isining girls (@ ys ) By (@ 15ye8 of « group of 16 persons is 28 yrs and 3 months, Two persons cach $8 yrs old let the group. The average age of the remaining persons is @ % ) 4 2 @ » ge age ofthe A30 Average, Problem on Ages 41, The average weight of & persons is increased by 2.5 ke Find the average of the frst 12 multiples of 12. When one oF them who weighs 56 ke is replaced by a new ( @ () 8% man, The weight of the new man is oO» @ ® (a) 73ke (b) 72ky, 4% The average of 30 observations is 45. If thre new (c) 75ke (@) None of these observations 42, 4 and 48 be added, find the new average. 42. A man drives t his office at 60 knvhr and returns heme fa 29 () 401 along the same route 30 km/hr. Find theaverage speed. © 42 (@ 4497 (a) SOknvir (b) 45th 47, Theaverage marks of 90 students is 56. Late on itis found (©) 40kmir (G) SStawhe that marks of one student is misread as 36 instead of 63 4B, One-third ofa certain journey is covered at th rate of 25 km Find the correet average. hr, one-fourth at the rate of 30km/ hr andtherestat SOkm (a) 562 &) 563 br. Find the average speed forthe whole journey. (©) 564 (@) Cannot be determined i ‘i 48. Whatis the average ofall numbers from 1 to 100 that endat (a) 335 km/hr (b) 44; km/hr 1 1 fo) 33h km tr (@ 44, km/br a ‘a 44, Find the average of first 40 natural numbers MG (@) 902 (b) 28. . (©) (@ 2s at o3 2 @s 1 (dd) u (dd) 21 @ 34 &) 4 (d) 2 ic) a fa) 22 | (@ 32 &) 42 c) 3 bets fw ts Tw fs Pe [3 Te +e ls [ms fel Te 1s le Ss fa) 15 {c) 25, (b) 35 «) 45 (a) él@lwlel«lwlsle! «| @ 7 be fr [ey fr Fe [7 Pe [ar To 8 (a) 18 {c) 28 ) 38. @ 48 (a) 9 fe fo Tm [2 Te [» Te Te Te 10. (d) 20 fc) | 30 (a) 40 (b) HINTS AND SOLUTIONS 1. (@_Letthe firstodd number, Abe x. 4 According to the question A+B+C+D+E=$%45 SNENIDIR HAE HOEK E8= 225 Required average 5. (a) Assume the thirdnumber =x According to question 280+%~ 1785 A=41, B=43,C=45,D=47 or, 5604 x4.387= 1405 Bx D=43%47=2021 of, x+917= 1405 2 (Average score (, x= 1405-917 = 488 566-+455-4231-+ 678498943: 6 (@) Required average - 7 965+ 362+18: 3976 568 8 3 (©) Supervisor's age =32+6=38 years Average, Problem on Ages 7 (@) Lethe first number be =6x Second number 3x and the third number =2x According to the question, Gxt Set 2x= 15403 er 1Ix= 1543, 1543 _ a= SS ar Required difference =6x—2x= 4x Avetage score = geet zt att 1 glea=a07 Let the five consecutive even numbers be x e+2,2+4,x+ Gand x+8 respectively Acvordin 0 oa diet ots+8 er 5+ 20-260 or $x 260-20 240 or x= 48 Sand E 2800 Box+2=48+2 X+R=4B+8=56 Be E=50% 56- Let A= x According to the question Dette n @ xtrtl be 5x48 5x4 10-240 S5x=230 x46 E=4644=50 Ax B= 46% 50-2300 12, (@) Letthe third number be=x 3x First number 3x and second number = — tothe question x=3«121 =3x121 Third number 66 Required diference Additional weight = 45-25 Bx—n=2x=2 x 66= 132 BR @ 20kg, Actual average weight =47 14, 16, (6), Third number (©) Sumot four int (©) Monthly income th (©) Averag ASL 290-(48. 107 =86 ers 73S «A= 204 (108+ 29)=187 2)-(535 *2) =290-97 Sum oftwomiddle integers =294 Difference between (worn! Is7—14 The smaller number = 15130 <4 146603 60320-42980 16540 (©) Let the seven consecutive integers be x6 Xx Lat The sum ofthe firstfive is xtxtlext2 +4=5x+10 sx+10 The average ofthese five is x42 of the seven will be of 1 results 1234567891011 Average of last 6 resulis= Average of first 6 results= 49 Itis quite obviousthat the sixth result is included twice, conce in the fist six results and second in the last six results Value ofthe sixth result = (Sum of first six results) (Sum oflast sx results) ~Sum of 11 results 60-20-9404 (@) Averageof20 numbers=0. Sum of 20 numbers= (0 « 20)=0. It is quite possible that 19 of these numbers may be positive and if their sum isa, then 20th number is (a. (@) Given A+B=40 o C+B=38 (i) A+C=42 ww O+G+ Gi) >A+B+C=60 iv) fiom (i) and (iv), we get C= 20 years B= 18 yearsand A= 22 years A32 ZB. @ 4 3. (b) 2%. tb) © Bb) a © 0. fa) nO) o Letthe number of persons, initially going for Picnie=x Sum oftheirages=16x Lox +1520 0, Iss Also, 5420 05x = 10> x=20 year Ageof the teacher = (37 «15 36 © 14) yea Let's” be theaverage seore aller 12h innings 12x 11x -2)465 xa Remaining pages = 512 -302=210 Letaverage printing error in remaining pages =» 1208+210%x 312 S210x=80=3x=4 Total runs=40 » 50= 2000 Lot hishhighest score be =x Then his lowest soore= x ~172 2000—x— (x=172. ow 8 Ne 38 = 26=2172-1904 ax=174 Letthe average after 17h inning =x. Then, average after 1th inning = (x ~3) Lo{x—3)+ 87= 17x or x=(87 ~48) Letthe averags 39. expenditure ofall thenine be Rsx x= 325 Total money spent=9x=9 «3. Then, 3 84x42 Rs.29.25 150% 30-135 416 30 Corrected! mean = Lot average cost of petrol per litre be Rs x 12000 Toor 4000 F000 ¥ Required average { %6. 0, I a a, Hence, new average Average, Problem on Ages Letthe other number beN. Y SN=2XY-X ‘Sum of 10 numbers= 402 Corrected sum of 10 numbers 13431 Is=402 402 Ifyou assume the first number to be a, naturally next ‘number would be 2 more than a and soon > Thenumbersare: a.a+2,a+4,a~6a+8, Hence, theiraverage = Sum/5 = (a+4) © © © © ©) @ © © of 28 krvhrand Total marks=45 * 502250 Average age ofthe re wining boys (15416 +2514) Ty x+dtx+ 6d x36 12-144 = 4e= 144-12 12 4 + Average age ofthe remaining girls 1050 ~(300+-400) 700 14 years 5=76k, By Direct Formula Let the total joumeybex km. Then * kivat the speed om at 3)0kawhr and therest distance eos = Bhs 50" 00) average speed = M0 33d mt oo 40d. (@ Sumof the firs 40 natureal numbers = = =820, 820 Average="5- Average, Problem on Ages Aas 45, () Sumofsthe first 12 multiples of 12 Correct sum of 90 students marks (12% 1)+(122)+ (12 «3) +04 (1212) 5040+ 63-36" 5067 = mC 2 soon ael2vel3 Comrect Average = “55-= 56.3, = DH12H13 _ 936 90 48, (@)Thenumbers from | 100 that eda 6 ar 6,16, 26,36 264 46,56,6,75,86,96 a8 Sum=6+16+26+ ..+96=510 4, (6) Sumof 30observations=45 «30= 1350 average= 22-51 “After adding three new observations sam 10 135042 +4448 = 1494 , abe, # @ let=2-£ New averags 6k, b= 10k, e-4k 47. (b) Sumof 90 students marks = 90 x 56= 5040 bic Ok+1Ok+Hke 20k Asmarks 63 misread 36 ¢ ak ae The word “percent” is derived from the latin words “per centurn” ‘which means “per hundred” A percentage isa faction with denominator hundred Iis denoted by the symbol Y Numerator ofthe fraction isealled the rate per cent Thaease vale Increase = IMETEASE vale op Original value Decrease value Decrease %6 = Perease val. gp Original vah the price of accenmodity increases by%, then reduction in consumption, so as not to inerease the expenditure is ( 100)" ioorr I the price of a commodity decreases by %, then the imercase in consumption so as not to decrease the 100 expenditure is | — * Uno: Population Formula: Ihe eriginal population ofa town is Prndheannual increase it", then the population after n years is P[1+—2-) and population before n years nog) =Eeen ¥ U*i00) I7the annual decrease be r %, then the population after n yenrs is P[I-—-) and 100) population boforen years 7 ‘00! First Increase and then decrease I the value is first increased by x % and then decreased by y% then there is x-y-22 Jug increase or decrease, according tothe +e 00) 8 or -ve sign respectively Percentage, Profit & Loss Successive inerease or decrease Ifthe value is increased successively by x% and y % then the final imerease is given by (x+9+55) the value is decreased suecessively by x% and y % then the final decrease is given by IfA’s income isr% more than that of B, then B's incomeis less than that of A by Torr +100 }% IFAs income is ¢% less than that ofB, then B's incomeis| ‘more than that of A by (cts) GZ! u Express the fraction “ into the per ent press e percent 12-100” 100 EEEEIASD 2: Rent ofthe house is increased from 7000 to® 7700, Express the ineresein price asa percentageof theoriginal Sol. Increase value =€7700-27000= $700 1 g,__ merase value 700 69-19 nerease 96> Oginal value 700 10% Percentage, Profit & Loss EXAMPLE RE certain eateper cent per annum. Now itis 456976, Four years itwas 390625, What wil itbe 2 years hence ? ‘Sol. Suppose the population increases at r¥ per annum. Then, The population ofa certain town increased ata ‘ ect) C.P,,then Profit =S.P. CP, > SP.=C.P,+Profit > CPSP. Profit So, Area isinereased by 29 : Ifthe sing price (S.P) of an article is greater than the nthe difference hetween the selling price and A385 ticle is less than the cast Loss : If the selling price (S.P) of an price(C.P), then the difference between the cost price (C.P.)and, the selling price (S.P.) is calle loss. Thus,ifS.P.» 100 loss, since it is ve, er} EEIUTZED 8: A shopkeeper sold two commosties for 800 earn and on one he gained 10% while on other he losses 10% how ‘much does he gain or loss in whole transaction 2 i ¢ ©) &% @ Sok (a) lowe ~ { SBMAR Hoss an gain)? _ 10)? at tos ( A (2 £36 10. Percentage, Profit & Loss ER QERCISE 11% of which number is 33? f@) 300 () 330 @ uo @ If two numbers are respectively 20! none of these and S0% of a third number, what is the percentage of the fist number to the second ? @ 0 2» ©» wo The digit a unit place ofa two-digit number is inerensed by 100% and the digit at ten places of the same number is 1. Thenewnumber thus formed is 19 more increased by 50 than the original number. What isthe original number? @ 2 0 6 © 4 (@) None ofthese Groundnut cil is now being sold at®27 per ke, During last month its cost was € 24 per kg. Find by how much % a family should reduce its consumption, so as to keep the expenditure same @ 115% 9 1 9% 9% © 16 oo (Chunilalinvests 65% in machinery, 20% in raw material and still has® 1,305 eash with im, Find his total investment f@) 76500 () 7,225 (©) R500 (@). None ofthese There is an inerease of 30% in the production of milk chocolates in Amul Dairy in one month. Ifnew it is9,LO0 milk chocolates per month, what was it one month ago? (a) 10,000chocolates (6) 9000 chocolates (6) 8000chocolates __(}-_7000 chocolates In a college elestion berween (ot 40% ofthe total votes polled, was defeated by his rival by 160 votes. The total number of votes polled was f@) 90 (bh) TH @ A number is increased by 10% and then reduced by 10%, Wo rivals, candidate who A ter these operations, the nurmber (a) does not chang (©) docreases by 19 (c) imereases by 1% —(}_ increases by 0.1% the price ofsugaris increased by 7%, then byhow much per cent should a howewife reduce her consumption of sugar, to have no extra expenditure? (a) Tover 10” (0) 107over 100% {o) 10Q0ver 107 ) % Ravi'ssalaryis 150% of Amit’s salary Amit'ssalaryis 80% of Ram's salary, What istheratio of Ram's salary «0 Ravi's| (@) 1102 () 203 @) 5106 @ 615 Incl, 4% ofthe bys same as ofthe gis and there are 20 girls, Total number of students in the class @ m7 Os Ws @ % An inspector rejects 0.08% of the metres as defective, How many metres will he examineto reject (@ 20m © (©) 500m @ Ifthe radius of a circle is diminished by 10%, the area is diminished by @ % ©) 20% (1% @ 10 In an election betwoen two candidates, 75% of the voters cast their votes, out of which 2% ofthe vates were declared invalid A candidate got 9261 votes which wer total valid votes. Find the total numberof votes enrol in that election (@) 1680) T?A’s salaryis 16800 (@) 18600 (@) 16008, higher than B's salary, then how much per cent is B's salary lower than A's” @) 22% 3 In the month of January, the Railway Police eaueht 4000 ticketless travellers, In February, the number rise by $% However, dueto constant vigil by the Police and the Railway staft fand in Apeilit further reduced by 10%. The total number of ticketless travellers caught in the month of April was; @ (b) 3255 (@) 3575 @ 40% ofthe people read newspaper X, 50% read newspaper Y and 10% read both the papers. What percentage of the people read neither newspaper? (a) 10% (by 15%) 2% @ 2% The length of a rectangular plot is increased by 25%. To keep its area unchanged, the width of the plot should be the number reduced by 5 3391 (a) kept unchanged (b) increased by 25% (6) increased by 20% — (d)_ reduced by 20% 1£90% of A= 30% of B and B=2x% of A thea the value of (@ 40 ) 40 © = (@) 190 AA reduction of 20% in the price of an appleenable aman to buy 10 apple more for & $4. Thereduced price of apple per dozen is (@ 2432 (b) 21296 (9) T1030 @ @440 The single discount which is equivalent to successive discount 020%, 15% and 10%is. @ 2% (by 429 (9) %62 (a) 383% Percentage, Profit & Loss 3. 3, the cost priceis 96% of the selling price, then what isthe proiit percent? (a) 45% bh) 4% © 4% @ 38% L Ain sold his book for Rs 891, thereby gaining => ofits cost price, Find his cost price. fa) E850 (b) E810) FEST (@ %40 A man buys 50 pencils for Rs 100 and sells 45 pencils for 90. Find hisgain or loss, @) 2m () 39 (c) 25% (@) Nogain or loss If 11 lichchus are bought for 10 paise and 10 lichchus are sold for! paise, the ) 10% (by 11% 20% @ 2% A shopkeeper purchases 10 kg of ice at €600 and sells ata lossas much theselling priceof? ky ofrice. Find the sale rateofrice/ kg. (a) ToOperks (0) &SDperky (o) €80perkg (@) 870per A dishonest fruit seller professes to sel his goods at the cost price but weighs 800 grams for a ke weight, Find his gain percent (a) 100% (by 15% (@) 50% @ 20 ‘A man sold two steal chairs for 500 each On one he gains 20% and on other, he loses 12%. How much does he gain or lose in the whole transaction? fa) 15% gain (6) 2% gain (0) 1.5% loss (@) 2% loss. Two electronic musical instruments were purchased for ©8000, The fist was sold ata profit of 40% and the second ut loss of 40%, Ifthe sale price was the same in both the cases, what was the cost price of two electronic musical (a) € 2000, 5000 (o) %2200, £5500 (0), €2400, % 5000 (@) %2400, 25600 A fuitseller sells 8 oranges ata cost price of 9. The profit per eeat is tl @ HE @ 5S @ 8 2 9 7 3 The cost price of 20 articles is equal to the selling price of 25 articles, The loss percent in the transaction is @ 5 (b) 20 © wv A single discount equal to a discount series of 10% and 20%6is f@) 2 (b) 28% —@ 30 Foracertain article, ifiscountis 25 the discount is 10%, then the profit is (a) 10% — (b) 20% =) 35% @ 3% A.wholesaler sels 20 pens atthe pri of 27 pens toarctile: The retailer sels the pens at their market price. The profit forthe retailer is 3 the profit is 25%, IF @ 1% © 1% © fa merchant estimates his profit as 20% ofthe slling price, ‘what is his real profit per cent? f@) 18% — (b) 29 6 (0) 25% @ 38. wy, 4 46, Aa7 The difference between the selling price ofa clock at aprofit of 8% and 10% is Rs 6. Find the cost price af the clock. (@ 7120 (by ZOHO) 7400 — @) F300 the numerator of fraction is increased by S00 denominator is increased by 300%, the resultant fraction and the is2. What was the original fraction? 4 2 o> o> , is » 6 oF @ What is 25% of SMof 5 rd of 630? (a) %S () 525 © 4 (685 The diference betwomn 58% of a number and 39% of the same number is 247. What is 629 ofthat number ? @ 130 () 806 © 754 @ 110 ‘What is240 per cont of 007 (1680 &) 17% © 16 @ 170 The population of atown is 198000. Itincreases by 7% in the Ist yoar and docreases by 5% in the 2nd year. What is the population oF the town at the end of 2 years (@ 211860 () 201267 (©) 22483 (@ 198900 The total number of boys in schoo! is 24% more than the {otal number of girls in the school. What is the respective ratfoof the total number oFboys to the total number of girls in the school? (25:31 (o) 31:25 © 91:21 (@ Cannot be determined Two candidates fought an election, One ofthom got 64% of the total vores and won with 992 votes, What was the total ‘number of votes polled ? (@ 3500 (&) 3580 (o) 3880 @ Twenty por cent of Anuj’s annual salary is oqual to seventy five per cent of Raj’sannual salary. Raj’s monthly salary is 60% of Ravi's monthly salary. 17 Ravi's annual salary is %14lakh, What is Anuj’smonthlysalary ? (@ %27000 (b) 27000 fo) %324000 (a) %5400 The price of petrol isinereased by 25%. How muich per cent ‘must a ear owner reduce his consumption of petrol so as not to increase his expenditure on petrol? (a) 25% () 5% © W% @ m Bysselling 12 notebooks, the seller ears a profit equal 10 the selling price of two notebooks. What is his percentage profit? (a) 25% ) 0% (6) Data inadequate A38 a ® ‘The radius of circle isso increased that its circumference 2. increased by 5 The area of the circle then increases by % () 102 (c) 105% @) 11.25% Milkcontains 10% water. What quantity of pure milk should be added to 20 L of milk to reduce this ta 40%? f@) 30k () 321 s (©) 361 @ ML In measuring the side of a square, an ertor of 5% in excess ismade, The error % in the calculated arca is, 1 fa) 104 34 ) 105 © ) 13% (25% a A dishonest dealer professesto sell his goods at cost price but he uses a weight of 960 g for the kg weight. Find his 55, inper cent, 1 a ) 42% @ 4 wy 4 (0) 9 @ 4% A dealer sold a mixer for €420 at aloss of price should he have sold it gain 12.5% (a) T620 () %s40 () 7650 (@ 750 Percentage, Profit & Loss Amilleman makes aprotitof20%on thesale of milk. Ihe wore 10 add 10% water to the milk, by what %ewvould his profit increase? 40 @ 9 o > © 2 @ 0 A man purchases two watches at Rs S60, He sells one at 15% profit and other at 10% loss. Then he neither gains nor loss, Find the eost price of each watch. (a) R8224, Rs300 (b) Rs200, Rs300 (©) Rs224, Rs336 (@)_ Rs200, R336 The list price of a watch is Rs 160, A retailer bought the same watch Rs 122.40, He got two suocessive discounts cone at 10% and the other ata rate which was not legible What is the second discount rate? @ 1 (b) 14% 1S! (a) 18% Asells atube {0 Bata profit of 20% and B sells it 10 Cat profitof25 % IFCpays Rs 225 for it, what did A pay for it? (@) 210 (b) 2125 (©) 150 @ 27 A man buys a single apple for & 25. Ifhe were to buy a dazen apples, he would have to pay a total amount of 250, What would be the approximate percent discount he ‘would get on buying dozen apples ? @ 2 (b) 2» © 2 @ 0 tLotuTot2 Tots Teta les Te 2 _@leflelzlols [ole Toles To 3 | @ |) os To Ts Te Ts Te 3s Ta Ts To 4 De [os To Ts a Te Tg ar To 3 @ [as Tw [2s To 3s To Tas Ta Ts To *“|@|«/olelols|@o|«|o] «| @ 7 Le | Pe [27 Pe [a7 | [a7 Te */o|e]|@ole2sl|eo)s| eo] «| @ 9 tos T@ [2 Tae [oe To fe Tao » Le [2 Te P30 Pa Te Te Ts Te INTS AND SOLUTIONS 1 (@) Letthenumber be x. Then 11% 0fx=33 3300 x= 230 300 11 4 (@)Letthe third numberbe 100, Then, the first and second numbers will be 20 and 50, respectively. Required %= 22 <100=40% (@ Working with options, we have Original = New Difference ber number @ 2 u 2 ) @ % 3 om 8 4 Obviously, (4) is the correct option. 27% .199 =, 2 (@)Yochange inrate For fixed expenditure,” “change in consumption change inmate 99 100+ % change in rate 10/8, 100 patit 5 +100 100) 1+! Percentage, Profit & Loss 5. (@ Lethe had originally® x. Then Is. 65%%afx+ 20% ofx + 1305=x 065x+02x+1305=x 90.15 x= 1305 =>x=88700 His total investment = 65% of 8700 +20 85% of $700=8 7395 Let one month ago, produetion be x chocolates Then, | 9100 of 3700 6 Ww 9100100 oO0chocolates 7 (b)_ Lettotal number of votes polled be x. Then, votes polled by other candidate (100-40)% of = 60% of. Now 60%afx~40%ofx= 160 20x * 100 Let the original number be 100, Then, the new number ~ 100° 1.1 * 0.999 ie. the number decreases by 1%, % reduction in consumption 160 800 votes a io Toor? 107 Letthesulary of Ram be € 100, Then, silary of Amit~€80and salary ofRavi~Z 12019, Ratioof Ram'ssalary toRavi's salary = 100 : 120 =5:6 m 1, (b) 40% orboys = 40% ofboys=10 girls Total no, of boys = 25 Total number of'sudents ~ 25 +20 Levthe inspector examined x metres, then 0.08% af x =2 2 © = 0.08 100 Ifthe radius is diminises by %, then RB © 10 10% 100 Arca is diminished by | 2r 2x10 100 14, (b) Let the total number oF votes enrolled be x. Then, Number of votes cast= 75% of 98% oF of). of [98% of (75% of'0)] id votes 9261 100" 100" 100°* a (92612100 100% 100 ({ 16800. 15x 9B IS A390 (©) LetB'ssalary be® 100, then A's salary = 125 125-100 99 lesser = L100=20" (@) Number ofticketless travellers in April (ss 00/100) (©) (A= 40, n(B)=50, (A > B)= 10, H(A U B)=n(A)+n(B)—n(A 9 BY =40-+50-10=80, Percentage reading cither or both newspapers 80%. Hence, percentage reading neither newspaper = (100-80)%=2 (@ reduction in breadth 20% (@ Ax90_Bx30 Too 100 > 3A=B (b) Let the original price of apple be® x /dozen fe SA S10 Now pice? = aonn = StS. 18 ).3 Hehe 1 = @ s ato Now, single discount of successive discount 32% and 10% 22419 2410 49 100 388 DB. (b) LaSP, =T100. Then, CP. =F 96; Proft=R4, it% ={ A100 Prats =| ag 100) () Lace, SP-cP, x then profi sxx =A891-x 10 x= 21H _g g19 1 24. (@) C.R forSOpencils~% 100 100, 45-8 90 0 CC. for 45 pencils S.P.of 45 pencils No gain , no loss 25. (a) CP. for I licheh ee 2. (@) CR for chen = piss P for | lichchu= 1 paise $.P for liehchu= 753 10 gain % = 1011 100= 21% # 10 i Lot S.B=xperke SP.of2keof rice Now, Loss=C.P.-S.P 2x =) 10x 350 perkg He gives 800 grams but charges the priceof 1000 grams (ke) sen every 800 grams, he gains (1000 200 grams, 2. (b) 2x z. ta) £800) grams ve og 2) " 300 His, 100% = 25% Short-cut : Gain’ 200 x 100 1000-200 (a) SPofthe Istchair= ¢ 500 Gain= 20% 500100 _ $00%100 ‘2 ate iat chair ~ 5002100 _ 3001100, 100720120 250 a @ @ © o © Percentage, Profit & Loss SP.ofthe 2ndehair= ¢ 500 Loss 12 500%100 _ $00%100 10-12 CP.orthe2ndehair $00%25 28025 _ 6250 2 i i Now S.P ofboth the chairs= ¢ 1000 CP ofboth the chairs 1250 , 6250 _ 19780-+18750 _ 32500 “yn ea Net gain = 1000 5 Gain a= 20083 100 » Gain 6 sp 59/39 32500 100 _20 15% (To oe place of decimal as (Toe place of decimal) Here, SP, =S Py 6 @, i 7 60cP, > = 400? 8000 = 2400 and CP, = 8000-2400 Let C.P of oneerange Then CP. of § onan; S.Pof$ oranges = 5600 Rel so gs 29 Gain then C.P. of: and SP. of 2 articles @ 20 100 jy 10220 Equivalent diseount= 1020 ~ “> =30-2=28% i site, 100-4 _ 100-4 gy nme 00d, 10g. 100. *To0=10 10+25 75 100g: 90 100+g, = 100+: 150 = gy Rowiler's SP, =M.P, Retailer's C.P. for 30 Pens =M P. of 27 pens. Retailer's S.P. for 30 pens = M.Pof30 pens 100 | 30-27 100 = % gain = Percentage, Profit & Loss %. (o Rew 1 profit % isthe profi % on CP. 2% profit on S.P, —*eprofiton SP. 199 100 —% profit on SP Real profit <100=2 ~ 100-20 3. (d) LetC.P.= 3 x.Then SP,-SP,- 26 0010) 008) io 10x 108x600 24-6009 € 300 (e) Letshe original fraction be Then, **=* 8 2 ay 7 Rw 0 2 630 =525 00 “7003 B. (@) Letthemumberbe=x Ascring tthe quogion (58-397 ofk—247 vo 997 247910044 ae ¥ 1300 9 root 300 = 1300-52-06 smootl0 io0 240 4, (o) 240% 0f700=700 742 = 1680 41, (6). Population of the tawn after 2 years 98000 [1+ 1 198000 T00 \100 J 19800010795 = 201267 100% 100, 42.) Let the number of girls inthe school be = Number of boys = a= amber of bays = was we Raquiedratio =x = 124: 100=31:25 4 M, 4 4% 4#, rere (Votes obtained by winner candids Votes obtained by loser candidate (100-64) =36° Difference of votes = (64 36) =28% According to question, Total votes = (©) Ravi'sannual salary= 1.44 lacs 14100000) Monthly salary 2 2 12000 @ Monthly salary of Raj= 12000 » {5 = 7200 Suppose annual salary of Anuj =x =7200«12« 00 then x Anujsmonthly salary= 324000 + 12= 27000 (@) _ -. Decrease in quantity = >> «100 190+25 25 100 = 20% Hence, owner must reduce 20% consumption of petrol (b) Let xe the selling price of I notchook Selling price of 2 note book = 2x= profit As, weknow, profit = selling price —eost price Cost price of 12 note book = 12x~2x= 10x 2% 100= 20% ox Profit percent (b) As the circumference increases by 5%, theradins also increases by 5% Present radius = 1,05 r We know that, area o (radius? Present area =(1.05)2 * previous area 1.1025 Previous are Percentage inerease in area = 10.25% 2x54 = 1040.25 =10.2 on| 25+] (@) Quantity of water in20L = 10%of20L Then, 04x ~ 100 > BOs 4e~ 200 > 4x= 120 = x= 30L rey ®. (a) If side is increased by 4%, area increased by as 50, (b) Error=1 kg-960 1000 960 g= 40.8 40 40 1 ‘ain Y%6 100 = 9 5 100= 3 1000-40 960719 85 51, (b) (100- Loss): $, :: (100+ gain): S, (100~ 12.5);420: : (100+ 12.5): 85 875:420. 1125:5, S)= 4200112. =s. 52. (b)_ Let profit perlitre=Rs20 So, CP litre=Rs 100 SBP./litre=Rs 120 On adding 10% water to thernilk 9 CP per 2 tire = Rs 100 0 SP. por © tie = Rs 120 ro 120%10 400 Rs SP perlitre = Rs 7S i) Woo, 40 by which profit increases = *-20 a © © @ Percentage, Profit & Loss Here, in wh 1, there is neither Joss, therefore Amount of gain in ene watch Amount of oss in other watch > 0.15xCP, =0.10«CP, | CR 0.102 os Also CP, +CP,=560 560 = Rs224 $60 -224=Rs336 re Retailer price= list price | |g (, 10), 2) 5122.40 =160{ 1 ° too I too 40100 _y 5 Too 160%90 2 ds =(1-0.85)100 Let A paid=Rsx 125% of 120%orx Cost ofone apple =& 25 Cost of 12 apples =25 « 12=% 300 Amount paid = 250 Discount= 300- 250-50 50100 Discount = (approx) Time, Work & Pipes, Cisterns Inthis chaper we will study the following two topics (9 Workand Time ii) Pipes and Cisterns * {fa person completes a job in n days then he will complete ~ tpartinone day. Above fac is clear from the followong examples: ETUIT) | : Ram is twice as good as Shyam in work; Shyam ‘ill do a pices of werk in 30 days; in how many days Ram will do thework? Sol. Since Ram is twice as good, he will do the work in == 15 days © IfAand B cando apiece of work in and Vays respectively While working alone, they will together take days om tocomplete it. EXAMPLE] can do same work in 15 days: in how many days ‘work together? Sol. As per the formula, required days hyam will doa pigee of work in 30 days Ram bath do the - == 10 days 30+15 45 ‘ @148,Cemdoapicet workin. 2days eect days tofinish it [EILIZG) 3 : shyar will doa piece of work in 30 days; Ram cando same work in 15 days, Bhuvan can dothe same work in 10 days; in how many days canal three do the work together? Sol. As per the formula, requir days 30x15<10 {303 +15%10+ 30610) 4500 300 Says © Ifaninlet pipe canfilla cistern in hours, the partfilled in © Ifpipe 4 is‘ times bigger than pipe B, then pipe A will 1 take <> ofthe time taken by pipe to fill the cistern. EET 4: tiakes rs for pipe d toempty a 100 liter tank; iffanothor pipe B which is double the size of pipe A is employed, how-long will ittaketo empty the tank? Sol. Since the Pipe is double the size, 1 itwill take + time ofthetimetaken by the smaller pipe 1 Therefore 54 =2 hrs © Ifxandy filvempty a cistern in ‘m' and ‘9 hours, then together they will ke aan ) Hours to fill/empty the cistern and in one hour“ th art ofthe cistern will be filled! emptied. (same as rime and work) FEMI 5 : There are to pipes (nk and outlet stached wih tank o 1000 ites, The nlt pipe can il the tnkin2 rs The outlet pipe can empty the tank in 4 hrs, What isthe time required to fil the tank in case both are open? In ene hou what par ofthetank wil be filed? Sa. For Inlet pipetime fil hetank=2hes For Outet pipe, ime toempty thetank = 4s Time ofilthetank 2x48 toiit Net par filled/emptied in onchour =~ 7-= th partofthe tank, which is obvious from the earlier result. was) 6 Time, Work & Pipes, Cistems ER QERCISE 26 men can complete a piess of work in 7 days, How many more men must be hired tocomplets the work in 13 days? fa) 9 (b) 8 (©) 6 (@ 18 Work done by'A in oneday is halffthe work done by Bin tone day. Work done by B is half ofthe work done by C in dane day. If C alone ean complete the work in 7 days, inhow many days can A, B and C together complete the work? fa) 8 ( 14 4 (21 émen can complete a picew af work in days. I how many) days can 12. men complete the same piece of work? f@) 10 (9 (@ Cannot bedetermined “A can complete a piece of work in 12 days. *A’ and *B* together can complete the same picce of work in 8 days, Ln how many days an “B alone complete the same piece of work? (a) 15 days (b) 18 da (©) 24 days (0) 28 days. 12 Men can complete one-third of the work in 8 days. In how many days can 16 men complete that work? (a) 18 (by 12 () 4 (G) Cannot bedetermined Computer A takes 3 minutes fo process an input while computer B takes $ minutes. Ifcomputers A, B and C can process an average of 14 inputs in one hour, how many minutes does Computer C alone take to process one input? f@) 10 4 (0) 6 @ 8 21 binders can bind 1400 books in 15 days. How many binders will be required to bind 800 books in 20 days? @7 (9 () 2 @ A andl Bean finish a work in 10 days whileB andl C can do itm 18 days. A started the work, worked for 5 days, then B worked for 10 days and the remaining work was finished by C in 15 days. In how many dayseould C alone have finished the whole work ? (a) 30 days (b) 15 days (©) 45 days (O24 days Ais 30% more efficient than B, How much time will they, working together, take to complete a job which A alone could have done in 23 days? (a) U1 days (b) 13 days (@ None of these 10, 16, Sunil and Pradeep ean complete a work in $ days and 15 days respectively: They both work for one day and then Sunil leaves, In how many days will the remaining work be completed by Pradeep ? (@) 1 days (b) 12 days (©) 15 days (@) 8 days Two pipes A and Bean fill a tank in 15-and 12 hours, regecively. Pipe ales kept open 3 ofthe ine and both pipesarekept open forthe remaining time. In how ‘many hours will the tank will be full? @ Ish () 20h (©) 10h (@ 13Sb 176men and Sboys can doa piece of workin 10 days while 26 men and 48 boys ean do the same in 2 days, the time taken by 15 men and 20 baysin doing thesame work will be: (a) 4 days 0) 5 days. (o) 6 days (@) 7 days. 12men completea work in 9 days. Afler they have worked far 6 days, 6 more men join them, How many days will they take to complete theremaining work? (a) 2 days () 3 days cys (@) 5 days. ps A, Band C ean fill tank in 12, 1S and 20 hours respectively ITA is open all the time and B andl C areopen forone hour eseh alternately, then the tank will be fll in (@) Ohrs, & 6 1 @ 75 hes (©) This, Two pipes A and B when working aloneean fill tank in 36 min. and min. respectively. A waste pipe C can empty the tank in 30 min, First and Bare qpened. After? min... Cis also opened. In how much time will the tank be full ? () 30min, (©) 20min, (@) 36min, A is half good a workman as Band together they finish a job in 14 days. In how many days working alone will B finish the job. (a) 20 days (0 2d (a) 39min, (b) 21 days (d) None of these Acan do ofa workin 12 days. In how many days can he finish | ofthe work? 8 (@) 6 days (©) 3 days () 5 days (@ 2 days Time, Work & Pipes, Cistems 18, 2. % Aman is twice as fas as a woman, Together the man and the woman do the piece of work in 8 days. In how many days each will dothe work ifengaged alone? (@)_man-t4days, woman-28 days (b) man-12days, woman-2 (c) man-10days, woman-20 days (@) None of these 2 men working 8 hours aday can finish a work in 10 days, Working at the rate of 10 hours « day, the number of men required to finish the same work in 6 days is f@) 30 (ty 32 © 4 (@ % A tyre has two punctures, The frst puncture along would have made the tyre at in 9 minutes and! the sscond alone would iaveclone it in Gminutes. fair leaks out ata constant rate, how long does it take both the punctures together to makeit flat? 1 1 (@) 1 minves (6) 3minutes 1 (0) 3 minutes (@ 4Lminutes ) a 4 3 “3 A man, woman ora bey can doa job in 20days,30 days or 60 days respectively. How many boys must assist 2 men and § women todo the work in 2 days? (a) 1S boys (b) 8 boys, (©) 10 bays (@ None of these If 1Swomen or 10 men can completea project in $5 days, in how many dayswill S women and 4 men working together complete the same project? @ 7 8 9 (@ A can doa piece of work in IO cays, while Balone in 15 days. They work together for S days and the rest of the work is done by C in 2 days. IP they get 450 for the whole work, how should they divide the money ? (a) £225,E150,275 (by 280,100, 100 (6) €200,2150,2100 (ad) 2175,2175,2 100 1 Aor working for 8 days, Anil finds that only = ofthe work has been done. He employs Rakesh who is 60% efficient as Anil. How many more days will Anil take to complete the job? (a) 15 days (b) 12 days (©) W days (8 days X can doa piewe of work in 15 days. Ifhe is joined by ¥ who is 50% more efficient, in what time will X and ¥ together finish the work? (a) Ways (b) 6 days (oc) 18 days (G) Data insufficient 1712 men ar 1$ women or 18 boys can dos piece of work in 15 days of § hours each, find how many men assisted by 5 women and6 boys will finish the same work in 16 days of9 hours each (a) 6men (o) Smen (by 2men (@ 4men », 3. 31 Ms 2. men and 3 boys can doa piece of work in 10 days while 3 men and 2 boys can do the same work in 8 days. In how many days can 2 men and I bay to the work ? dys i} ‘Amman can doa piece of workin 5 days but with the help of his son he ean do itn 3 days In what ime ean the son do italone? 1 ; (a) 85 days (b) 7 days 1 @7 (@) 8 days PipeA can fill tank in Shours, pipe Bin 10 hours and pipe C in 30 hours, [Fall the pipesare open, in how many hours, will the tank be filled ? @2 ) 25 @ 35 Two tapscan fila tank in [2 and 18 minutes respectively Both are kept open for 2 minutes and the first is turned off In how many minutes more will the tank be filled ? (a) 18min (©) Amin, () 20min, (@ Bmin One fill pipe is 3 times faster than second fill pipe B: takes 10 minutes less ime to fila cistern than B takes. Find ‘when the cistern will be full f fil pipe Bis only openee. (@) 20min (6) 18min (©) 15min (@ 10min Three fill pipes A, Band C can fill separately a cistern in 3 4 and 6 minutes respectively. A was opened first After 1 minute, B was opened and after 2 minutes from the start of A.C us also opened, Find the time when the cistern will be full? 1 @ 25min (by 4 min (9 33min (@) None of these 4 Water flows at 3 metres per sec through a pipe of radius 4 cm. How many hours willittake oflla tank 40 metres long, 30 metres broadand 8 metres deep, ifthe pipe remains ful? (@) 176.6hours (&) 120hour (©) 135.5hours (@) None of these 40 men can cut 60 trees is hrs. If8 men Jeaves the jab how ‘many trees will be cut in 12 hours? @ 2 o) 0 8 (@ % 10horses and 15 cows eat grass ofS acres in a certain time, How many acres will feed 15 horses and 10 cous for the same time, supposing a horse eats as much as 2 cows ? (b) 398acres (a) 259 acres (@) 40/7acres (e) 40/11 eros AAG 36 x, 41 4 pipes can fill a reservoir in 15, 20, 30 and 60 hours respectively. The first was opened at 6 am, second at 7am third at am and fourth at 9am, When will thereseroirbe fall? @) Wan (b) 12pm. (©) 1230pm (O 100pm Twopipes can fill acistem in 14 and 16 hours respectively. The pipes are opened simultancously and itis found that dduc to leakage in the bottom, 32 minutes extra are taken for the cistern to be filled up. IFthe cistern is fill, in what time would the leak emp Hobe (b) 112he (o) Shr (a) 100br A tank is filled in 5 hours by three pipes A, B and C. The pipeC is twice as fastas Band B istwice as fastas A. How much time will pipe alone take to fillthe tank? (a) 2hes (by 25hes (o) 35hrs (@ Cannot bedetermind 12 buckets of water fill a tank when the capacity of each tank is 1.5 litres. How many buckets will bemecded to fill the same tank, ifthe eapacity of each bucket i 9 litres ? 8 (b) Is (0) 16 (@ 18 A cistern has a leak which would empty itin 8 hours. A tap is turned on whieh adits 6 litres a minute into the cistern and it isnow emptied in |2 hours. The cistern can hold (a) 7860 tres (by 6840litres (©) 8640 litres (@ 8840litres Rekha can do. piece of work in 40 days. Surekha is 25% more efficient than Rekha, The number of days taken by Surekha to dothe same piove of work is (a) 24 days (b) 28 days (o) 32 days (@) 35 days A can doa work in 12 daysand Bin 15 days. Ifthey work ca it together for 6 days, then the factioh of the work that is leftis 1 1 a) + b= @ 5 ® 5 1 ©) @ io 4 4s i 4a #, Time, Work & Pipes, Cistems A alone can complcte work in 15 days and B alone in 20 days. Starting with A, the work on altemate days. Thetotal work will becompleted in @ 17 days (©) 16 days (0) 4 days (@) 13 days Alone can complete a work in 12 days and Balone in 48 days. Starting with B, they work on alternate days, The total work will be completed in @) 18 days (b) 19 days (@) Data insufficient 4 men candoa piece af work in 10 days, 2 women can doit in 15 days and children ean doit in [2days. In how man days can 8men, $ women and |S children together complete the piooe of work? (a) 2 days ) 3 days (©) 4 days (a) None of these 48 workers can reap a field in 9 days, Ifthe work is to be completed in 6 days, the extra workers required are (@ % o © @ 2 4 men and 6 boys ean doa pieccof workin 8 days and 6men and boys can doit in 7 days. Ifthe daily wages of a bay be 20, what will be the weekly wagesofa man? (a) €280 () &275 (© 270 (@ 26: A is thrice as good a workman as Band is thereforeable to finish apiece of work in 30 days less than BB Find the time in Which they can do it working together (a) 45 days () 11% days (©) 18 days (@) 22% days ‘takes tice as much time as B to do a work. Working together the work iscompleted in 8 days. can do the work alonein (a) 24 days (©) 16 days: (6) 12 days (d) 28 days Time, Work & Pipes, Cistems nar a @ [Tu T@ 72 utol« le 6 | 2 | w@ [2 @ [3s Tw fs 9 @ A RE ST Rien 1 nv o © © Days Men 1a 26 so 131 xv where is thenumber ofmen required to complete the workin 13 days 6x 13x= 1726 1726 5 Number of additional men =34 Ratio of work efficiency of A, Band C 1:2:4 Ratio of time taken tofinish the work Bd:2=4:201 Timetaken to finish the work by Balone 34Men 26=8 ©) = 1 2= 1d days Time taken to finish the work by A alone Tx d= 28 days work done in | day by A, Band C crs So time taken to complete the work by A, B and C together = 4 days 6 men ean complete the work in 8 days, ‘man can completethe work in 8% 16 12:men can complete the same work in %® &) 168 cays. Number of days = =24 days 12-8 12 mencan complete the whole workin 8x3=24 days 12x24 Required no. of days = Part processed by computer A in | minute Part processed by computer B in 1 minute Part processed by computer C in 1 minute 2 “60 5 42-20-12 10 1 wo a0 Hence computer C wil proses Input in6 rime. 80052118 Mes 1400 20 Let C completes the work in x days Work done by (A+) in I day Workdoneby(B+C) in | day= is AA's Says? work + B's 10 days’ work +C’s 1Sdlays? work= | or (A~B)’s S cays! work + (B+ €)'sS days’ work ' 10 days? work = 1 55,10 1 "i018 x X45 days Ratio of times taken by A and B = 100: 130= 10:13, Suppose B takes x days to do the work Then, 10: 13::23 299 33 A’s 1 day's work lo l 3p B's L days work= 355 Time, Work & Pipes, Cistems (A+ BY's1 day's work= A.and B together can complete the job in 13 days. Sunil iakes 5 clays and Pradeep takes 15 days to do the work 20 Part fled in6 hrs = | 3* 55 |= Ret wa(1-17)-3 nF 20)” 20 Let the required time to fil the tank be x hours Accrington Now tether and Band 2, pr a by 3 2) x x}+4{s-4x)=1 ‘And B in | hour, 4 a Toa time then to Fill the tank . = (6+ I)hrs=7hrs = =! 16 0 48 x= ib loure (0) Part fille in 7 min, = 7 Let | man’s I day's work =x and 1 boy's I day's work = y Remaining pare= [1-35] = 55 Then, 6x + 8y= and 26x 48y Part filled by(A+B+C) in | min. i0 2 Solving these two oquations, we get (tay 4 1 -(s6*5-30)* oo 100 ™¥* 300 1 part ln 60 min (15 men +20 boys)'s I day's work 12 aan sow! & pat fitin = 6023 = 39min, 1s 20) 1 20 20 {00300 16 (b) Let Bean dothe workin days ‘and A can do the work in 2x days 15 men and20 boys can do the work in 4 days, . (given) Liman’s 1 day's work = 5p L = x=2x14=21 days 12 men’s 6 day's work= {+6 By Direct Formula Remaining woek =| 1 Time taken by B= 14{ +1 \ + 3 8 men’s I day’swork= ("x18 = 17, @) + Acamdo Sof the workin 12 days 18 men's day'swork= { Zxts) = 2 1 Aan do + of thework in 12x days 2 days work is done by them in 1 day 8 38 ‘ 18 (&) _Letthe man alone do the work in xdays. Then, the woman alone do the workin 2x days. 1 work is done by them in 6x L he Theirone day’s work = 5th part of whole work men complete the remaining workin ~3days man in = 123 days 1233 (12+6) men in= 2 days ‘man takes 12 days and woman 2x =24 days Time, Work & Pipes, Cistems a9 1 (b)_ mpxdy xt, xw. 24x 108 « 1=m, m, xd, xt 3 All the three pipes together will fill the tank in hours ‘ 1 Pat filled by frst tap in one win =" ah rr Part led by ssond tapi one ain = Fh unfilled part = 1 B = tied pant = th 50 © 2 @ 3B. @) M @) %. (a) x. ( mb) 1 | iy partoftankis filled by sscand tp in Imi, 18 ® ° +h oft fi tyson tp int min, 3 = 18% min = 13min Let Ban fill the cistern inx min. Thea, then A.can fill thecistem in Given x—¥=10> x=15 min Leteister wil be fillin x min, Then, part filled by Ain x min + part filled by Bin (x part filled by Cin(x-2)min= 1 ymin % © 19x pa 2gmin Radius ofthe pipe (f)= 4 em=0.04 meter Volume of water flowing out per see nr? x rateof flow %. 72 5.0.04? x3 eu meters = 0.0151 cubic m 30 o0i51 Time taken tofill the tank =40 0. 40x30%8 1, oo “3600 M, = 40, D, =8 (As days and hrs both denote time) W, = 60 (cutting of trees is taken as work) M,=40-8=32,D,=12,W,=? Putting the values inthe formula M,D, W,=M, D, W, Weave ,40 x 8x W 321260) 4x8 horse=2 cows, LO horses = 20 cows. = Whorses + 15 eaws=20+ 15 = 35 cows, 15 horses + 10 cows = 40 cows. Now 35 cows eat $ 76.6 hours, 1260 onW 4. <, 40 40 =40conseat 5» 35 = Here we have converted! everythin you can work in terms of horses also, Lt the time be t hours after 6 am, in terms of cows, 1p ty 20 H+2u-2 2) 3) oO 3)-00 Its fill at 1 pr 1 is 434 t= Thours 0 Time, Work & Pipes, Cistems ister filled by both pipes in ane hour La “i i6 na Both pipes filled heeistemn in 1? ha, Now, dueto 1 bath pipes filled the cistern in 12 2 Lsns o efilled part in one hour = = part ofcistern emptied, dueto leakage in one hour would empty the cistern. kes x hours to ill the tank. Suppose pipe A alor Thes; piper and C'wil ake: 2 and % boars repel he tank 24244 35 his. Capac ofthe tank (12 135) ives~ 162 ies Capeatyofeech bucks ives 16 nner arncesnntea~ (2) 1 Jn 1 hour, empty part = 5th When tap is turned on, then ‘empty part in I hour= th Partofeistern emptied, due to leak 13 Uhour = $= = 32 =the Now, ln I min, cistern fil= 6 lit 1 in hr, cistern fill= 61 ng fil= ori Cistem can hold = 6 « 60 » 24 litre = 8640 itre Ratio of efficiency of Rekhe and Surckha = 100 :125=4:5 Hence, ratioof time taken todo.a work by Rekha and Surckha= $:4 So, if eka ca doa work in 40 days, Surekha wall do the same work in 32 days, 9 Warklett =(1 10 Time, Work & Pipes, Cistems ASL 4%. (©) In 9days work complete by=48 workers 8. (a) (A+BY's2day’s work =75 +55" Go In| day work complete by=48 9 $89 Work done in 8 pairs of days =| In 6 days work complete by= ~~ =72 workers 1 The extra workers required = 72-48= 24 workers, @ 47. (@) 4M+6B=8days > 32M +48 B= I day ‘ Also, 6M + B= 7 days Work done by Aon 1th day = 7 = M+28R- Tay Total timetaken = 17 days SO RMHSB=AM > 1M=20B “ = IM-2B Wages ofa man =2 20= 840 15 of 40 *7ie, 8280 por week 16 48 (0). Ravioof work doneby and Bin the sme time=3 1 ( Ratio oftime taken byA and B= 1:3 Remaining work =|! is ‘Suppose B takes x daysto finish a work. Then, A takes (30) days inh it (ted Remaining wok =( 2-2 ]= 3 x30 x3 Work done by Bon 19th day = = i-S=n t > x=45days Now, [5 workisdone by Ain I day, Thus, And B can finish the work in 15 days and 45 ' dys respectively. g Works doen by Ain 2s ‘Now, (A+ B)’s | day's work = tried \ ont oka aE as Tota be take 2: last So, both together can finish the work in 5 1 4s, \ 48, (@) Amon's | day's work = days=11days @ ys work = days atl a 4. (a). Ratio oftimes of Aand B=2: 1 nd B= 1:2 man’s I day'swork = 0 => Ratio of work dane by A 51 day's work = (A+ By's 1 day's work woman’s | day's wor! z J woman’ day’s work = 35 Divide 7 inthe ratio 1:2 ‘S children’s | day’s work = A’s | ds k + «chiles day’swork = 2 Hence, A alone can finish the work in 24 days oo Now, (8 men +S women + 15 children)'s I day's work 85.18 1.1.1 37 w 3060 5/6 4 60 60 So, they ean finish the work in days The rate at which any moving body covers a particular distance is called its speed. _ Distanes Speed Speed > time SI unit of speed is metre per second (mps). It is also ‘measured in kilometers per hour (kmp) or miles per hour (mph, Conversion of © Thour= 60 minutes: 1 km= 100m Tkm=0.6214 mile 1,609km, i.e, Ske 3 feet 60 60 seconds. = I mile Smiles 1 yard Total Distance Average sposd Total time fa certain distance (d), say from A 10, is covered at kur and thesame distance is covered again say from Bro A in *b’ knwhr, then the average speed during the whole journey is given by 2ab ) kaw Average speed = {5 | Ga (which isthe harmoniemeans of and b faperson with two different speeds U& V cover thes distance, then required distance Ux u-v Difference between arrival time XV Also, rived distance = Total time taken 2 Time, Speed and Distance FEIIGD 1 : Atoy waking sta speed of 10 kmh reaches his school 12 min lt, Next timeat a speed of 1S knvh reaches his School 7 min ht, Find the titan his schoo rom his house? Sol. Difference between thetime=12~7=Smin == Le 2 Roquited distance = #210, F 15-10 "12 Aman leaves a point A att, and reaches the point B att, Another man Jeaves the point B att, and reaches the point AAat t, then they will meet at lz) (to - )+(tq~ts) Tine taken by a tran to rosea pla san _ Lensth of train Speed of train Time taken by a train to cross platformibridge ete, (ie. @ stationary object with some length) __ length of train + length of platform bridge ‘speed of tain When two trains with lengths L, and Land with speeds S, and S, respectively, then (@) When theyare moving in thesame direction, time taken by the faster train to cross the slower train ——— th __, “ifference of their specds () When they are moving in the opposite direction, time taken by the tains to cross each other ~ sam oftheir speeds Suppose two trains or two bodies are moving in the same direction at wknwhr and vkrwhr respectively such that u> then their elative speed = (uv) kmh If their lengths be x km and y km respectively then time taken by the faster train to cross the slower train (moving in =) hs the same direction) = Time, Speed and Distance Suppose two trains oF nwo bodies ate moving in opposite directionsatu kavhr and v kea/hr, then ther relative speed (+ ypkmhe theirlengths bex km &y km, then ETI 2 : How long docs a train 99 m long running at the rate of $4 kh take to cross (b)_ platform 120 m long! (4) another train 160'm long running st 36 knvh in same direction? (9 aman running at 6knvh in same diretion? (b). Theplatform is statienary of length = 120m, Length to be covered Length of the train + Length ofthe platform 90+ 120= 210m Required time see (@) Another tain is moving in same direction. Length to be covered = Length of the train =90 + 160=250m Relative speed = 54 length ofthe other train 36= 18 kph, Required time = 2% 0 se 16 ix (®‘Thomanis moving in same direction, so Length to be covered = Length of the train, and relative speed = speed of train ~ speed of man Required time = °° (4-On= 90227 nae ‘ofboat with the stream (or downstream or D/S) (X+Y) ave. Speed of boat against the stream (or upstream or U/S) (X-Y) mie FETT) 3 : A bout is rowed down ariver 28 km in 4 hours and up ariver 12 km in 6 hours. Find the speed ofthe boat andthe Sol. Downstream speed is 28 7 kmph Speed of Boat = | (Downstream + Upstream Speed) 4.5 kmph Speed ofeurrent = (Downstream-Upstream spay 25 kph EEE 4: 4 man can row 6 krvh in still water. When the river is running at 1.2 kin’, ittakes him I hour torow to. place ancl back. How far is the place? Sol. Man's n=(6+12) Man’srate upstream ~ (6 ~ 1.2) kv Lethe required distancebe x km 7.2kwh, 48 kaw, ate downs Then = 10rd. & svan rows 27km with the stream an! 13km agains the stream taking 4 hours each time. Find this rate per hour sill waterand the rate at which the stream fw. se Spet wire steam 22-63 jmp 4 Is 3 Specs against the stream =F =3- kph Specs! of the man in still water Lap jie 1.Skmph Speed ofthe ssam= 5{ 6; Ast 6 Time, Speed and Distance ERQERCISE A man walking at the rateofS km/h crossesa bridge in 15) minutes, The length ofthe bridle (in metres) is fa) ao (b) 750 (c) 1000 (a) 250 A cyclist covers a distance of 750m in 2min 30 see. Whatis the speed in kin/h ofthe eyelist ? (a) 18km/a (b). 15k (c) 20km'h (@)_ None of these An aeroplane flies along the four sides of a square at the speeds of 200, 400, 600 and 800 km/h, Find the average speed of the plane around the fel (a) 384k (b) 370kmM (©) 368krvh (a) None ofthese Amanvatsacaaindsascanresusin 64 He it would take woride can walk both ways in 7, How Ik both ways in 7. How both ways L () 4S hous (42s @ Ghous There are 20 poles with a constant distance between each pole. A car takes 24 seconds to reach the 12th pole . How much time will ittaketo reach thelast pole? fa) 2525s (b) 1745s ©) (a) 4145s On a journey across Bombay, a tourist bus averages LOkrw far 20% of the distance, 30 kwh fr 60% ot and 20 kmh for the remainder. The average speed for the whole journey was (a) 10kntn (b) 30kmh () Skmh (@) 20kmh Ina $00 mrace around a stadium havi F200 m, the top runner meets the last runner on the Sth minute ofthe race, IFthe top runner runs at twice the speed ofthe last runner, what isthe time taken by the top runner to finish the race’? 20min (b) 15min (e) 10min (@) Sein A man walks half ofthe journey at4 km/h by eyeledoes one third ofjourneyat 12 knvh and rides the remainder journey ina horse cart at 9 km/h, thuscompleting the whole journey in 6 hours and 12 minutes. The length ofthe journey is 1332 fa) 36km km (a) 36k 0) (©) okm (a) 28m Rand § start walking each other at 10.AM at the speeds of 3 kmh and 4 kwh respectively: They were initially 17. Sk apart, At what time dothey meet? (a) 2:30PM (b) 11:30AM (6) 1:30PM (@) 12:30PM 10. 16, fl walk at 4knvhb, I miss the bus by 10 minutes, Ifl walk at Skaw/h, Ireach 5 minutes before the arrival ofthe bus. How far I walk to reach the bus stand ? (@) Skm (b) 45km © 3 km/h (Cannot be determined Without stoppages, a train travels certain distance with an average the same distance with an average speed of 60 km/h, How ‘many minutes per hour the train stops ? @ 15 ©) 18 @ 0 @ Ifaman valkstohis offceat 3/4 ofhisusual nite, he reaches office 1/Sf an hour laterthan usual. What is his usual time to reach aifice spool of 80 kwh, and with stoppages, it eovers None of these @ th ©) Ih © 7h (@) None of these ALtrain running between two stations A and Barrives at its destination 10 minutes late when its speed is SO kaw and 50 minutes late when its speed is 30knvh. What isthe distance between the stations A and B ? (@) 40km (®) S0km (© km (@ Dkm A thief goes away with a Maruti car ata speed of 40 km/h. The theft has been discovered after half an hour and the owner sets off in another car at $0 kmh, When will the owner overtake the thief from the start. 1 (@ 2Shours © 2hr 20min (© thra5min @ A long everytlay. Histimi cannot be determined distance runner runs 9 laps of a 400 metres track. (in minutes) for Four consecutive days are88, 96, 80and’87 resploctively. On an average, how many rmetrevminute does the runner cover ? (@) 40nvmin (©) 45min (©) 38nvmin (@ Bavmin In. flight of 600 km, an aircraft was slowed down due to had weather. Its average speed forthe trip was reduced by 200 kn hr-and the time of ight increased by 30 minutes, The duration of the Might is; (a) Lhours o (©) 3hours (@) 4hours A thiefstealsa carat? :30 pam.and drivesitat @ kmph. The theft is discoveredat3 p.m. and the owner sets off in another carat 75 kmpl. When will he overtake the thief? (@ 4:30pm (b) 4:45pm. (o) Spam @ 5:15pm, Time, Speed and Distance 18 20, Poinis A and Bare 70 km aparton a highway. One carstarts| form A and the anather one fram Bat the same time. Ifthey travel n the same direction, they meetin 7 hours. Butif they travel towards cach other, they moet in one hour. The speeds of the two ears are, respectively fa) 4Sand25 km’ (b) 7Oand 10knvh (©) 4and 30km'n (@)_ 60nd 40 kv A train covers 180 km distance in 4 hous, Another train covers the same distance in 1 hour less. What is the difference in the distances coveral by these trains in one hour ? @) 45km o (©) km @ A river 3 mdeep and 40m wideis flowing peer hour, How much water (in ites) will alLintothe sea in a (@) 400000 (b) 40.0090 (©) 40.000 (4900 The speed of a boat in still water is 1Skm/h and the: mis S kh, The distance travelled downstteam in 24 km None of these x the rate of 2km eof kn (b) 8k (c) 6km (a) 16m A man makes his upward journey at 16 kanyh and downward journey at 28 km/h, What is hisaverage speed ? (a) 324mm (b) Sokmvh (©) 2036kmm () Pkmh Acartravelsa distance of 75 km atthe sposd of 2Skm br. It covers the next 25 km ofits journey atthe speed of km/hr and the last S0 km ofits journey atthe speed of 25 kan/hr What is the average speed of the car? (@) 40kan/he (b). 2Skmhr (0) 1Skaw/he (4) 12.5kmbe A car travels. distance of 5 kmsat the speed of 15 kmph. It covers the next $0 kms ofits journey at the speed of 25, kkmph and the last 25 kms of ts journeyat the speed of 10, mph. What is the average speed of the car? (a) 40kmph (b) 24kmph (6) 16kmph (4) 18kmph The ratio betwoen the speed of atrain and acar is 18 13. Also, a bus covered a distance of 480 kms, in 12 hours. The speed of the bus is five-ninth the speed of the tran, How auch distance will the (@) 2s0im. (b) 28okm. (6) 20km. (4) Cannot be determined Train A crosses a stationary train B in 35 seconds and a pole in 14 seconds with the same spood. The length af the train Ais 280 meters. What isthe length ofthe stationary train B? (0) 360 meters (6) 400 meters ar cover in S hours? (b) 480 meters (&) 420maers 2. ASS ALtrain covered a certain distance ata uniform speed. I'the train had been 6 knv/h faster, then it would have taken 4 hours less than the scheduled time. And, ifthe train slower by 6 km/h, then the train would havetaken 6 hours more than the scheduled time, The length ofthe jouneyis (@) 700km (b) 740km (9) 20km (@) 7@km On a journey aeross Bombay, a tourist bus averages 10 krvh fe 20% ofthe distanee, 30knv/h for 60% oft and 20 km/h for the remainder. The average speed for the whole Journey was (@) 10kmi o) (© Skmh @ Aman rides @ horse at the stops for S min to changehorse at the end of every seventh mile, How long will he take to caver distance of 96 miles? (Approx) (@) Thr 20min (b) 6hr.25 min (©) 8hr.42min (@) 9hr.48 min Arman starts from B to K and another from K to B at the 30kmh 20 km utSSe of 11 miles an hour, but same time, After passing each other they complete their 4 journeys in 3-5 and4 = hours, respectively Find the speed ofthe second man ifthe speed ofthe first is 12 km/hr, (a) 25kmph (b)1Okmph (6) 12.66krph (@) 20kmph The driver of a car driving @ 36 kmph locates a bus 40) meters ahead ofhim, After 20 seconds the bus is 60 meters bochind. The speed of the bus is (a) 2%6kmph (b) 20mise. (o) T2mésee. (@) 18kmph Atrain overtakes two persons walking alonga railway track. The first one walks at 4.5 krw/h, The other one walks at 5.4 kav The train neods 8.4 and 8.5 seconds respectively to overtake them, What is the speed of the train if both the persons are walking in the same direction as the train (@) km (b) 72kmm (©) 78kmis (@) 81imwh Aman covers acertain distance on a scooter, Ifthe scooter moved 4 knvbh faster, it would take 30 minutes less. IF it moved 2 kinvh slower, it would have taken 20 minutes more @ () sskm (©) sskm (@) s0km Rand§ start walking towards each other at 10. AM at the spoods of 3k and 4 knvh respectively, They were initially 17.5 kmapart. Atwhat imedothey meet? f@) 2:30PM () 1:30AM (©) 1:30PM (@ 12:30PM A56 Time, Speed and Distance 23 |e). 251) INTS AND SOLUTIONS 6 @ @) @) © @ @ speed (5+ 5) m/sce=38 m/s Distance cover in 5 minutes Spest= 18) em/he-=18 km be 5 Leteach side ofthe squarebe x km andlet theaverage spoad ofthe plane around the field be y knw, Then, xXx x dk 300 400 * 600" 800 -y 25 3400 4x Average speed 384 kwh, We know that, the relation in time taken with two different modes of tansport is Koa bah fide ban™ 2 (bya FE) 4 19 3 SLT is a4 a Let the distance between each pole be x m, Then, the distance up to 12th pole = 11 xm Speed Time taken to covers the total distance of 19x 19x24 Tix Letthe average speod bex km/h, and Total distance = ykm, Then, = 4145s 02,06 02) _y 10? 30°" a0" x sxe n2mm/h O05 10, © @ @ @) @) » Aer $ minutes (before meeting) the tp runner covers 2rounds i, 400m and the last runner covers | round ie,200m Toprunner cavers 800 m race in 10 minutes, Let the length ofthe journey =x km. Journey rides by horse cart = x{1 1 km, 6 1 Then, total time taken tocomplete journey =—hF Sh 3 x= 31,216 36 2m = 36km Let after thours they meet then, a+4t= 1751 Time= 10am +25 h= 12:30pm : difference of time = product af speed] erence oF time z speed “ference of speed | 45/10 eo | 5 = d=5km Duc to stoppages, it covers 20 km less Time taken tocaver 20km=22 = “ 7 | [Here, -ve sign indicates a= before the schedule time] 16min 15min 4 uml imex{®1)) change in time Time, Speed and Distance 18. (b)_ Let thedistancebetveen the twostations bex km Then, X10 50 7 6 06 Thus distance between the station A and B = 5 ke 14 (@)_ Distance to be covered by the thief and by the ewner Letafier time , owner catches the thief Total distance 5. a) Average speed = Bf . Total time 400449 40049 B+96+89+87 360 metres minutes 16 (a) Letthe duration ofthe fight be xhours. Then, 600600 _ 55, 6001200 _ 544 x ya! Sx (2x13 28 +x-3-05 xt HOR 1)=0 Sx- Ihr [neglecting the -ve valuc of 17, (e) Here, distance to be covered by the thief and by the Letafter 2: 30 p. m., owner catches the thiefin thrs, sts Shvs (3 Then, 60» t=75| t=) So, the thiefis overtaken at $ p.m, Let the speed of the cars be x knvh and y km’h, respectively Their relative spoods when they are moving in same direction=(x—ypkrvh, Their relative speeds when they are in opposite directions=(x + yp kia, Now, <1 or x+y=70 and = 7 or x-y=10 i) -y Salving (and Gi), wehave x=40 km/h andy=30 kmh 1Skm 19% (@) Required difference AST 20, (b) Volumeof water flowed in an hour 2000 » 40 3 cubic metre~240000 cubic metre volume qf water flowed in | minute 240000 4000 cab metre = 40, 00,000 litre © $= 20m. 21. (b) Downstream speed= 1S Required distance = 20 km @ Time taken to cover first 75 km of distance 18 FoF hours Time taken o cover next 2Skm ofistance Time taken t cover last SO km of its journey Isokm 1Ohours Total imetaken =3 Average speed mph, 10 Let the distance travelled during both upward and downward journey be x km, ‘ gu Tos distance co erage speed = ~Toal time taken 2 28x16 4 xXx 2816 1628 28516 Timi taken to covera distanecoF45 kms 4. © 3hours, Time taken to cover distance of $0 kms 50 Time taken to cover distance of 25 kms “10 Total distance= (45 +50 =@+ 25) kms = 120 kms Total +2,5)hours=7.5 hours 120 Required average speed! 16kmph 73 Speed ofbus 40 kmh Speedoftrain= 40% 2 =72 kine Speed of ear = “x13 =52 kane Distance covered by ear=52 * $= 260km. ASS 2% (a) 2 © 2a 2 0. 280 Speed of train A = = = 20meter/seeond L 2035-280 meter thottrain B. 700-280 meter =420 meter Letthe length ofthe journey be xkm. ‘Suppose speed of the train be y kak * hours Time taken to coverx km: Solving these equations, we get y=30,x=720, Length ofthe journey = 720km. Let theaverage speed bex kav. ‘and Total distance = ym. Then, 02 06, Time taken totravel96 miles 96 pss During the journey of 96 miles, he has to stop for 13 times to change the horse Total stoppage time <5 mins, = land $ mins, Shrs and 43 minutes Heenee the total time = Shrsand 43 mins + brand S mins his andl 48 mins, Ist man's speed 2nd man's speed Time, Speed and Distance 2 6 2nd man's speed ~ 5 a4, i= % —tonm sspoat= © = toxmm: (@_Net distance gained by car over the bus 40+» 60 100m, in 20 see Distance Time Relative speed =m 18kmph, 5 ig &5Akmh=(54-| misec=1.5inises is Let the speed ofthe train be § mse. Then, (S 84=(S-15)x85 28.48, 1278 > 08 1.28) 105-8. 22535: cpetotitnin-(225%!) nh km Lethdstcehoxkim Lt potas x x 30 a @ oo 20 i) (On solving (and (i), Putx=3yin (i) we get wy yea Hence, distance = 20% 3 = 60km. Let they meet after t hours, then, +4 4 @ 115 3t=25 2:30pm, Time=10am +25: eT | Interest isthe fixed amount paid on borrowed money. The sum lentis called the Principal The sum of the principal and interest is called the Amount. Interest is of two kinds imple Interest : When interest is calculated on the original principal forany length of time, itis ealled simple interest. Principal» Time Rate ‘Simple interest, 7 PXReT ie SL 10 Interest Amount= Principal ie ASPeL=P+ 00S. Principal (p) = 100*S! xT ate) = OO*SE xP Timecr = 10st PaR Irate ofsimple interest lifes from yar to yer, then ARy Ry tad 100 FEUUEDD 1 Find in intrest tobe paid ona oan 26000 st 5% per year for 5 yas Sol. P=%5000, R=5% and T= years SL=P: PxR&T _ 6000x55 gu, = PERET _ 000% "> ~z1500 100 100 FETLIZd 2 : Find the amount to be paid back on a loan of 18,000 at 5.5% per annum for 3 years Sol. P=818000, R=5.5%, T=3 years Pex 5.53 5 -PxRHT _1000%5.5 = 100 100 Amount P-+ 1= 18000 + 2970 220970 Simple Interest & Compound Interest EEILIZGp 5: tn how many years will a sum of money tiple itself, at 25% perannum simple interest, Sol. Letthe sum of money be P. So, A= 3P andS.L=A~P=3P-P=2P WooxS _100%2P PRR Px25 = (i) Compound Interest : Money is said tobe lent at compound interest wien at the end of a year or other fixed period, the interest that has become due is not paid tothe lender, but is, added to thesum lent, and theameunt thus obtained becomes theprincipal in the next year or period The process is repeated tunitilthe amount for the last period has been found. Hence, When the interest charged after a certain specified time periad is added toform new principal x the next time period, the interest is said toe compounded and the total interest acorued is compound interes, cu=Pi(t “Too! rate of compound interest differs from year to year, then Amount (A) = P{ sera = (u Alu sle a Since ris calculated half-ycarly therefore the rate per cent will become half and the time period will become twice, ie., ‘Compound interest — when interests calculated quarterly Since | year has 4 quarters therefore rate of interest will become thofthe rate of interest perannum, and the time period willbe 4 times the time given in years Hence, for quarterly interes A-60 FSTMI 4 : Vind the compound interest on 225625 tor 12 ‘months at 16% per annum, compounded quarterly. Sol. Principal (P) ~%25625 «4-16 16% =" Rato(t) Time~ 12 months~4 quarters 25625(14 4) =25625| 00) } 256252526, 26,28 _g0677.62 2 25 CL=A-P=29977.82 35262 Difference between Compound Interest and Simple Interest ® Casi i CL-Su When gy PR?/ 20048) @ cosine af ® i Ca-sh= 7 Loo) Simple Interest & Compound Interest EXHIB) 5 : The difference between compound interest and simple interest on a certain amount of money at 5% per annum, for2,yearsis®15. Find thesum (@) %4500 () %7500 (9) %000 (@ 000 Sol. (@) Letthe sum be 100, 10052 Therefore, SI Rs 10 100 and ci =100/1 -100 00 = 100% 2221 jogs 4 20x20 4 Difference of C1 and St ==2-10=+ 4 If the difference is d ‘the sum = 100 = If the differene 400 15 = 86000 oR is 15, the sum 15100100 Thesum is= = 26000. Simple Interest & Compound Interest x AL ER QERCISE (On what sum of money lent out at 9% per annum simple interest for 6 years docs the simple interest amount to 8107 fa) 7900 (©) %1200 () T1000 @ ts00 1 in what time will € 72 become 81 at 6% pa, simple Inwh il € 72 become 81 at 67% pa. simp interest? (@) Lyear6months (6) 2 years 1 (©) 1 years (25 years nd 1 L Out ofa certain sum, 5 rdlisinvested at 3% that O% thorestat 8% Ifthe simple interest fo 2 years froma these investments amounts to& 600, find the original sum. (a) 4000 (b) 5000 (o) 6000 @) e700 A sam of money doubles itself in 10 years at simple interest Inhow many years would it triple itsel? @ 10 @ 15 ©) » @ 3 The interest on sum of money at the end of 2 years is (a) 10% (2 () 6 @ 2% 1 Thesimpleintereston a sum ofmoney is 5 ofthe principal and the number of years is equal tothe rate % p.a, The rate A man buys landand pays20 timesitsannual rent. What is the rate %? (@) 2” fo) 10 @ A sum of money ent out at simple interestamounts to& 720 after 2 yearsand to 1,020 aitera further period ofS years Find the sum and the rate (a) 500, 10% (b) F600, 10% (o) €500, 12% (@) F600, 12% } 9. On retirement, 2 person gets 1,53 lakhs of his provident find whieh he invests in a scheme at 20% p.a, His monthly income from this scheme wil bo f@ 72,450 () 72,500 fo) 2,550 (@) 22,600 The simple interest on & 200 for 7 months at $ paise per rupee per month is ( %70 ) 7 (© 35 (@) %3050 A father left a will of& 68,000 to be divided between histo sons aged 10 years and 12 years such that they may get cqual amount when cach attains the age of 18 years Ifthe money's reckoned at 10% pa, find how much: ch gots at thetimeorthewill (@) %30,000, 38,000 (&) 7 28,000,240,000 (©) %32,000, 236,000 (@ cannot he det Ifthere are three sum of money RQ and R so that Pis the simple interes of Q and is the simple interest of R rate’ and time are same in each case, then the relation oF P, Qand Risgiven by fa) P-QR (b) Q=PR © R=PQ (@) POR=100 ‘What annual instalment will discharge adebt of 4,200 due in years at 10% simple interest? (@ %500peryear — (b) 600 peryear (©) © 700peryear (a) F800 per year A lent & 5000 0B for2 years and %3000 te Cfer 4 years on simple interest at the sa ne rate OF interest and Feceived % 2200 in all om both of thems intros, The rate of interest peransum is: @ % © @) 10% The difference between the simple interest received from two different sources on & 1500 for 3 yoars is € 13.50. The difference between their rates of interest is: fa) 01% ©) 0. © 03% @ 0 An amount of 1,00,000 is invested in twotypes of shares, The first yields an interest of 9% p.a. and the second, 11% p.a. Ifthe total interest atthe end of one year is 95% then pa. Ifthe total interest atthe end of tb invested in each share was: 500 (by 62, 500; 37,500 500 (a) 82, 500; € 17,500 A62 o. 2. A person invested in all® 2600 ar 4%, 6% and 8% per annum simple interest. At the end of the year, he got the sume interes in all the three cases, Themoney invested at 4% is f@) 200 (b) 600 (o) 300 @ F120 IfRe. | produces 29 in 60 years at simple interest, the rate © 4% 15% @ What sum of money lent out at compound interest will amount to% 968 in? yearsat 10% pa. interest bing charged annually? fa) 600 () 700 (©) 800 (@ F900 the differencebetween S.1 and C.1 fo yearson a sumor ‘money lent at 5% is © 6, then the sun B, (a) 72200 () 72400 (6) 2600 (@) 2000 What will 1000be werth afer three years fit cams interest ‘compounded annually 2 a) T1075 ) 71287 z11s7 @ %300 A sum of money placed at compound interest doublesitself in5 years Find in how many years it wil come 8 times. fa) 1Syears (b) 20 years re) @ wy The C.L. on acertain sum of money forthe 4th year at 8% p.a.is@ 486. What was the compound interest far the third year om the same sum at the same rate? (a) % 450 ) R47, (©) 436 (@)_ None of these The difference between compound interest and simple interest on a certain sum of money in 3 years atthe rate of 7% per annum is® 225.645. What is the principal” f@) € 10,900 ) © 15,000 (©) € 17900 (@) % 20000 atthe rateot S 25 years 2. Simple Interest & Compound Interest The simple interest accrued on an amount of” 19,800 at the nd ofthree years is ° 7,128. What would be the compound. Interest accrued on the same amount at the same rate in the same period? (a) %9934.6784 () Ts0175744 (©) 27861. (@) Cannot be determined ‘What isthe interest received on a principal of 450 for 2 yea ifthe interest receivedon | afer four year atthe same rate of simple interest is€ 0.407 @ i () Z180 © B36 (@) Cannot be determined Mr. Deepak invested an amount of 21,250 for 6 years. At wh 26,350 at theend of years? @ 4 © Sudhanshu invested 15,000 at interest @ 10% p.a forone year. Ifthe interest is compounded every six months what amount will Sudhanshu get at the end ofthe year? @ %6s () %165000 31652550 (@ F810 ate of simple interest will he obtain the total amount of pa ) S%pa 8%pa @ 12%pa © The simple interest accrued on an amount of $4,000 a the end of three year is€ 30,240, What would bethe compound interest acerued on the same amount atthe same rate in the same period? 230013 (b) 73191395 34013, @z Thesimple interest accrued on a sum of eertain principal is & 1,200 in Four ye the simple interest acerued on thrige ofthat principal atthe @ © 3.01395 atthe rate of 8%p.a. What woul be rateof'6% pain 3 @ ts () T3025 (@ 2.250 @ 22,150 Simple Interest & Compound Interest Ce Tae Pe Ts Tg To Ty Ts oy Tw To) Riese @ o o © @ ) Oy} o a3 zo p= 281004 1 a6 Wa SiLintowe=2P-P=tP ‘Now, $1. in Tl case = 3P—P=@ a OO, 20 years SxP “ 9 “Letannul rent 201K RL 100 Silo foe 5 years Principal =% 720 Now, P=600,T=2,81.=120 120x100 20052 120)=2 600 2 ZL, then principal (1020-720) = 300 =Rs 120 201 1 6 2 [wot Toe Te Te Tr oe 72 Tae Ta To 3 To Ts Te [3 Te Tis (io Ts Ta os To + tLeots Te ts Tw Te To [Te [2 To = l@lele ls le 20 | [2s Tm To | 2 1 @ © 2. (b) 3. ©) 4 @ Lsaxt0"20) 100 Rate=5 paise per rupee =5% Let$1,-®% = 200557 To Let one gets =& x then, second gets = 2 (68,000 x) Given :A, =A, 4p E108 _ 6g ggg xp + {58000 =n) 1056 100 100 = x{100 +80) = (68,000 x)[100 +60) _ 1808 10 x = 68000516 =x 36,000 68,000-% 32,000 Qurat yg Rerat and 100 Q 100 P_Q_m QR 100 Q=PR. Shortout method It borrowed amount be® Mand it istobe paid in equal instalments, then rain) Maur ioon¥ Hore, M4200, y= 110.0 5,02 04 5151 anon = su}, SED io = 200 =a [541] = 6 = 4200 =a=8 700 Le the tebe R% pa. The, (30003. R x2 0 200 (soon) 100) > 100R +120 2200 10, x20 | 220 16. i Oy} o @) } 00xRyx3)_ (1500, x3) 1350 To) 18 + 4st Ry «130-9 RyRy = 22 Let the summ invested at 9% be® x and! that invested at 11 be & (100000 ~x). then, (22242) f 00000) st] 00) 100 100000x +100 1100000 11x 100 Sum invested at 9% Sum invested at | 1%= & (100000. 62500) =€ 37500, Lattheparts box, yand [2600-(x +), Then, axdvel _yx6x _[2600-(0-+y)}x8x Too 100 100 ¥o4 2 ory we gt s.\ 600-Sx)u8 so, Xxx 3) 100 700 ty = ROO SE 8 505 = (7800%8) 78008 2 x={ 228) 200 Money invested at 4% = 1200 HereS.A-29,P 8 _ 810041 7x60 968 =P[t+ 2°] = p- 25841010 _ e599 100 TixT Difference = PA T0000 PxSus => 6 400 = 22400 70000 cert 100 ‘A= Amoutt P Principal 1 =rate of interes Required amount = 1000 222) 100) 1000 2h 2h 2h 1000 * 1.157 =F 1157 ~ 20° 20°20 ~ 7 2%, @ o) © co) Simple Interest & Compound Interest Let p(t 2) gp w00) (RY os fk oir By =2 =H (+i00) -2°-]U*i00] | men R\ ay safe BY a(t) one i00) “Goo Ife bee imret fh year, then 108% of x= 486 2486100 2 450, 108 #00028) Difference fin am ('a00+7 => msgrs= Px BOD 7100) 256%5:100:1005100 =P st 15000 pate IHS T2810 5, Principal (Area of base) (stant height) Lateral surface | * height — Right circular nth 2mih+0) arth r= Radius cylinder — | | — b=Height bi prisen Perimeter ofbase | 2 (area ofbase)+ | Area arbase ~ » (height) lateral surfacearea) height Cone ml » b R= Radius A68 Elementary Mensuration Name Figure Lateral Curved Total surface Volume ‘Nomenclature Surface Area Area Sphere : 4 An zm = Radius: Hemisphere ; am? aa (2) r= Radius Frsun ee, +e) a(n +1) Elementary Mensuration M69 ER QERCISE |. Find thearea ofa triangle whose ices ane'50 m, 78m, 112m respectively and also find the perpendicular ftom the opposite angle on the side 112 m, 20m () 30m (©) 40m @ 6m The side and the height ofa rhombus are 13 and 20 ems Find the area B respective! fa) 2600 6) (c) 29am" (@)_ None of these The circumference of aeitele is 44 metres. Find the area of thecirele fa) 154m (0) 160m (©) 178m? (168m? 4 Thelength and breadth ofa rectanglear itsarea is 720 m?, find its perimeter (a) 112met (©) 110 metre in the ratio 9: 5.1 () 11Smetre (D118 mete 5. How many squares are there in a $ inch by 5 inch square rid, ifthe grid is made up one inch by one inch squares ? @ 90 ©) 150 (o) 3 @ 3 6 Mtheratio ofareas oftwosquaresis9 : |, theratioof their perimeter is fa) 91 34 (©) 3:1 @ 13 A circle and a rectangle have the same perimeter. The sides eare 18 em and 26 cm. Whatis thearea ofthe ofthoreetang Sen? (6) 1Stan (©) 128bem™ () 616en & Ifthe perimeter and diagonal of rectangleare 14 and S ems respectively, find its area (a) em? o) (6) 20cm’ (@ 2en 9, When thecircumference and area ofa creleare numerically equal, then the diameter is numerically equal to (a) area (b) circumference © 4 @ 2x 10, In a parallelogram, the length of one diagonal and the perpendicular dropped on that diagonalare 30 and 20 metres respectively, Find ts area (a) 600m? () 40m (©) 680m (@), 574m 1. The area of a triangle is 615 m?, If one ofits sides is 123, mete, find the length ofthe perpendicular dropped on that side from opposite verte, (a) 1Smetres ©) (0) 1Ometres @ 2. How many plants will be there ina circular bed whose outer edge measure 30 cms, allowing 4 cm for cach plant ? f@) 18 () 750 © 4 @ 220 12metres None of these B 1, 2, square earpet with an aren 169m? must have2 metres cute off one of its edges in order t0 be a perfect fit for a rectangualar room. What isthe area of rectangular room? (a) 180m? () 16n? (©) 152m (@ 143nF Ifthearea of circle dersases by 36%, then theradivs ofa circle decreases by (a) 20% ©) 8 © 2% @ o% The altitude drawn to the base of an isosceles triangles ® em and the perimeter is32cm. Theares ofthe triangles (@) Dow (6), soem? (©) 66cm? (@)_ None ofthese Thearea of square field is $76 km?, How long wll take fora horse to run around atthe speed 12 km/h ? (@ 1h (©) 10h (@ &h @ 6h Four cqul circles are described about the four corners ofa square so that each touches two ofthe others. Ifa side of the square is 14 em, then the area enclosed between the cirounferencesofthecielesis @ ow co) (©) Sten? @ The ratio between the length and the breadth of a rectangular park is 3:2. [faman eyeling along the boundary ofthe park atthe sposd of 12km /hr completes one round in8 minutes, thon the area ofthe park (in 9, m) is @ 130 (18340 (9 x70 («507200 wire cam bo bent in the form ofa cece of radius S6 cm. I itis bent inthe form ofa square, then its area will be @ ©) 00cm? © (6) 8800cm? The length of a room is double its breadth. The cost of colouring the ceiling st %25 persq, mis ® 5,000 and the cost intng the four walls at 240 pee sq mis 63,800. Find the height of the room, @) 43m (35m Atal cube of sonallereubes. Ifthe oa and 8 em, then find the edge of the third smaller cube, 420m 196em2 &) 4m @ Sm 1¢ [2em s melted and formed into throe sof two smaller cubes are 6 cm (a) Wem (b) 14em (em (@ 16em A well 22.5 deepand of diameter 7m has to becuz aut. Find the cost of plastering is inner curved surface at 83 per sq (a) t146s © tars (b) E1ass (@ %1495 «70 ® %0. The length, breadth and height ofa cuboid arein the ratio 33, 1:2:3. The length, breadth and height of the cuboid are increased by 100%, 200% and 200 the increase in the volume ofthe cuboid will be s respectively, Then, (a) Stimes (b) times (©) 12times (@) 17Kmes x. The surface area of a cube is 150 m2. The len diagonal is (@) sim () 5m lo A @ ism x The length of the longest rod that ean be placed in a room which is 12 m long, 91m broad and 8 m high is @) 27m @) 19m (©) 17m @ Bm the volume of a sphere is divided by its surlace area, the result is 27 ems, The radius ofthe sphere is 2%, (a) Soms (©) 27ems (6) Siems (@ 23cms The volume of water measured on a rectangular field $00 m 300m is 3000 m, Find the depth (amount) ofrain thatlhas fallen () Jems (c) 4oms (@ 3.Sems How many spherical bullets can be made out of a lead 37 oylinder 28 em high and with base rudivs 6 em, each bullet being |.Semin diameter? fa) 1885 () is (©) 12 (1782 Water flows oat through a circular pipe whose internal diameter is 2 em, atthe rate of 6 metres per second into cylindrical tank, the radiusof whose base's 60 cm. Byhow 38, much will the level of water tise in 30minutes? @) 2m ) 4m () 3m @ The capacity ofa cylindrical wnk is 246.4 hres, Ifthe height is 4 metres, what isthe diameter of the base? f@) 14m (b) 28m %. (©) 14m (None of these A reservoirs supplied from a pipe 6 cm in diameter. How many pipes of 3 ems diameter would discharge the same quantity, supposing the velocity of water is same ? @ 4 ©) © 6 7 40, the radius ofa sphereis increased by 2m, then its surface area increases by 352 em? The radius of the sphere befare the increase was' f@) 3a fo) Sem () 4am @) 6am (oy @ | 16 Elementary Mensuration Ahollowsphereofintemnal and external diameters 4emand 8 em respectively is melted into a cone of base di cm, The height ofthe cone is (@) Bem (b) em () Sem (@) 18cm Thereare two concentric circular tracks ofradii 100 mand 102 m, respectively. runs on the inner track and goes once roundon the inner tack in I min'30 see, while Bruns on the outer track in | min 32sec, Wha runs faster? (@) BothAandBareequal (b) A © B (@)_ None of these Thelength and breadth of aplayground are 36m and 21m respectively. Poles are required to be fixed all along the boundary at a distance 3m apart. The number of poles required will be @ » ) ® oO 7 @ 0 Acroctangular plate is of 6m breadth and 12 m length. To apertures of 2m diamoter each and one apertures of 1m diameter have been made withthe help of agus cutter. What is thearea of the remaining partion ofthe plate? (@) 68.5. m. (©) 62.5sqm (oe) 645sq.m (a) None of these aur sheets Sem * S em are arranged witheut overlappin to form a square having side 55 cm, What is the area of inner square so formes!? (@ 28000n? () 202 (©) 100m? (@) None of these A garden is 24 m longand 4m wide. There isa path 1 m wide outside the ganen along its sides. Ifthe path isto be constructed with square marble tiles 20 em * 20 em, the ‘numberof tiles raquired to cover the path is (@) 1800 200 () 200 @ 2190 2m of rain has fallen on a sa, km of land. Assuming that 50% of the raindrops could have been collected and contained in a pool having a 100m 10m base, by what level would the water level in the pool have increased? (@ ism ) 20m (©) 10m @ 3m Thelength ofa rectangular fild is double its width, Inside the field there isa square-shaped pond 8 m long. ihe area of the pondis 1/8 of the area of the field, what isthe length ofthe field? @ 2m (9m © @ 16m 20m 26 © @ @ [7 [wm [2 (oy [27 (by 20 Tia | 25 1s_| i | 23 | o Tio [2s | 28 Tia [2 To [3s Elementary Mensuration INTS AND SOLUTIONS 6 (b) Horea=$0m,b=78m,c=112m soso m112)=12m s-a=120 s-b= 120 s-c= 120 50=70m 78=42m 112=8m ROX TOXADER = 1680 sq.m, ‘Area Arca = base x perpendicular 2Arca 1680: Base 112 (a) Arca of shombus = side » height Perpendicular = 30m, = 13 x20=260 en (a) Inacirele, circumference Hence, 44=2nr “4 Now, (@) Let the length and breadth of a rectangle are 9 * m and $ « m respectively. Ina rectangle, area=length x breadth 720-94 5x or P16 xn 4 Thus, length = 9 4=36m and breadth =5* 4 =20im Therefore, perimeter of rectangle ~ 2(36+20)= 112m (Required no, of squares (©) Letthearea oftwo squares be x and x respectively. So, sides ofboth squares will be Px and J respectively. [sinee, side = Now, perimeters ofboth squares wil be siesOx and 44 respectively [since , perimeter 4 side] ANT ave 2(08+26) Thus, ratio oftheir perimeters = (Pes 2 er ofthe circle 88

+216+512=1728 16, (©) Area of field =576 m2, Then, > x3 1000-2x= 10 cach side offield = VS76 = 24m Thus the edge of third cube = 106m. Dina Giveiae wr eNae 2. (b) Area ofthe inner curved surface ofthe well dug =Pevimeser of square field =244=96km distance _ 9 Time taken by horse ~ © * =8b Total ast = 498 (@) Latte length wreath and height of the cuboid bes, & @ edie epetes ° TN Three wimax 21% 3062? ( Sie mages beaaih a scotty (CC ) Th ee CUNY VA Incresein volume 102%" _, Original volume 6x" pedi = 2x, 6x and 9x, The shaded An gives the required region. ah @) Inacuba, Area =6 (side)? or 150=6 (side)? of the shaded region = Area of the square area of four quacirants ofthe circles =c4y- aed site= 5 5m z a Length of diagonal = 3x side = SDB == 649 196 1M Boe 25. (¢} Required length = length of the diagonal 18, (b) Perimeter=Distance covered in 8 min, Vistas ote aay 81464 = Y280 =17 (22222) m= 160 m 2% Inasphere volume Lot length = 3x metresand breadth = 2x metres, Thhen,2 (3x+2x)= 1600 or = 160. Length = 480 mand Breadth = Area= (480 320)mn?= 153600 m and surface area = 4? According to question, Sr = or 1=27%3=81 ems 19, (¢) Lengthof wire (@ Let depth ofrain beh metce. Then, volume of water =area of rectangular field» depth ofrain 100 $00» 3008 Sidleof thesquare em=88 em, 3000 3000100 500%300"" “500x300 Area ofthe square = (88 » 88) em?=7744 em? 2%. (a) Letthelength of the room be ¢ m Thenits, breadth= //2 28, (6) Nolumeofeylinder= (626 *28)em? = (36 » 28)remr (4 £5000 Volume ofeach bullet Elementary Mensuration Bom Volume of cylinder Number ofbullets= oy nc sch bullet (aox2myn if a © =30 «60% 7 5 (1)? «(600) = 60h=30* 600) > h=300cm=3m 30, (4) Volume ofthe tank= 246.4 litres =246400 cm, Let the radius of the base ber om. Then, xr «400 | =246400 2 _ (2461007 ar -{ AAT) io6>r=14 a0) Diameter ofthe base=2r=28 em =,28m 31. (a) Nomberof discharge pipe lume of water supply pi ‘lume of water in each discharge pipe 2 @ o(e+2 +1)e+ 2-1) =28 22rt2=14>r=6om (b) Volume of material in the sphere ight of the cone be h em. sedaich Fuse yrs) 4x56 a Radius ofthe inner track = 100m, and time loem, uO) min 30 see = 90 see Also, Radius of the outer track min 3 Now, speed af A who runs on the inner track 102m and time: ec = M2sec 2x(100) 20% 8 698 ¥6. © i) © M8 And speed of B who runs on the outer track 2n(02) _ Six 30 23 = 6.96 Since, speed of A> speed of B ‘Avuns faster than B. Given, playgroundis rectangular. Length = 36 m, Breadth =21 m Now, perimeter ofplayground=2(21 + 36)= 114 Now, poles are fixedalong the boundary ata distance a1 14 Required no. of poles = —~=38 Given, Length= 12m and Breadth = 6m Aron of rectangular plate~ 12*6= 72 m2 Since, two apertures of 3 m diameter each have been ‘made from this plate, Arca of these two apertures = a(L)? +x(1)? Toalarenofaperne= 2x-+ = "B= 2° = 7 Area of the remaining portion ofthe plate 99 909 “ 20 a.m = 64S sq.m wt Side of the inner square = $5 Area of inner square=45 » 45 24mand Given, length of garden breadth of 14m ‘Area ofthe garden = 24 « 14.2 = 336m Since, heres 1 m wide path outside the ganten ‘Area of Garden (including path) =(24+2)x (14+2)=26 16m? =416m2 Now, Areaof Path = Area of garden inculding path) Areaof Garden = 416-336-8007 Now, Arca of Marbles = 20 x 20=400cm? Area of Path Marbles required ~ Soc op Marbles Volume of rin that is to be collected in a poo! 2x bo” 1 $= 10! em=104 meter Volume ofPool = L« Bh 101005 10h ot Toox10 Elementary Mensuration Let width ofthe field= bm lengih=2 Now, area of rectangular field= 26 b= 2 Area of square shaped pond =8 « 8 =64 According to the question, 2) =p bE = 6d > b= 16m length ofthe field = 16» 2= 32m i PERMUTATION Each of the arrangements, which can bemade by taki all ofa number of things is called PERMUTATION, For Example: Formation of numbers, word formation, sitting arrangement in a row. Fe eae te) The number of permutations of w things taken r at a time is denoted by "P.. It is defind as", = —! on "P= nl ular Pern Arrangements ro The number of circular permutations ofn different things taken z all ata time ig “= (1) 1 if clockwise and anticlockwise orders are taken as different Arrangements of beadsor lowers all different) around accircular necklace or garland: The number ofeircular permutations of different things tak all ata time is +(n- 1), ifelockwiseand anticlockwise orders are taken tobesame, COMBINATION Each of the different selections that ean be made with a given ‘number of objects taken some or all of them ata time is called a ‘Combination. The number of combinations of n dissimilar things faken r at a time is denoted by"C, or Cl, ).Itis defined as, Aa REMEMBER LMR =rtG, Permutation, Combination & Probability 1 =o (nr tt" CONDITIONALCOMBINATIONS Number of combinations of distint things taking. ( ) 6 © x @ In single throw with four dice, the probability of throwing 4 » 3 @) oo 6 6 16 20 oz @a 6 6 There are four machinesandit is known that exactly two of thom are faulty: They are tested, one by onc, in a random order till both the faulty machines are identified. Then the probability that only two tests aro neoded is » @ o jt Wy Three integers are chosen at random from the first 20 ers, The probability that their product is even, is © @ % os » 2 d oz o> A dics loaded in such a way that each ead number istwiee al likely to occuras each even number. IfE isthe eventofa number greater than or equal to4 ona single tss ofthe dic, then P(E) is 4 @ 3 0) 1 l @ > o 2 3 The probability that two integers chosen at random and their product will have thesame last digivis oF @ 3. Permutation, Combination & Probability Seven people seat themselves indiscriminately at round table The probability that two distinguished persons will be next to cach othr is 1 l @ ® > © 4 @ 5 IFtwo squares probability that they havea side in common is are chosen at random on a chess board, the i 5 @ ¢ ) z q og w 2 A speaks the tut in 70 percent cases and in 80 percent The probability that they will contradict eaet. ather when describing a single event is @ 036 ©, © 04 @ 038 042 The probability that 4 can solve a problem is solve itis . IFboth tempt the problem, what is the probability that the problem eis solved? = ) 12 an @ © w 2 Course materials are sent to students by a distance education institution. The probability that they will send a wrong program's stidy materials There isa is damaged in transit, probability of > that the packs P a P and there is probability of + that there isa short shipment. What is the probability thai the complete material for the course arrives without any damage in transit’? 8 a) o) & « ° @ 8 4 o @s A tug contain 5 white, 7 red and 8 black balls 174 ballsare drawn one by one with replacement, what isthe probability thar all are white ? 1 @ 3 w 2 256 i os @ z Permutation, Combination & Probability 38. Abag has 4 red and 5 black bulls, A second bag has 3 red and 7 black balls, One ball is drawn from the first bag and two from the sesond., The probability that there are wo black balls and a red ball is @ 4 oo 5 5 © @ 2 is u Two dice arc tossed. The probability thatthe tral score is a prime number is 1 5 @ © 6 a i * @ © 38. Abagcontains3 whitealls and? black balls. Another bag contains 2 white balls and 4 black balls. bag is taken and a balls picked at random from it. The probability that the ball will be white is ©) 7 x0 @) © @ in 5 36, Abag contains 2 rod, 3 groon and blue balls.2 balls areto be drain randomly: What is probability that the balls drawn contain no blue ball? s 10 @ o ic) x, 4%, a A fir coin is tossed repeatedly. Ifthe tail appears on first four tosses, then the probability of the head appearing on the fifth toss equals @ Suppose six coins are tossed simultancously. Then the probability of getting atleast one tail is @ ) 3 3s L © © Four different objects 1, 2,3, 44 are distributed at random in four places marked 1,2, 3,4. What is the probability that none of the objects oocupy the place corresponding to its number ? @ i o 2 © @ 2 Thereare 6 positive and 8 negative numbers. Four numbers are chosen at random andl multiplied. The probability that the product isa positive number is 500 503 a) ) © S01 Toor 0s 11 3) @ © Too ‘oot 80 ( wotutotwlotal@ ots, e ob) Permutation, Combination & Probability L 6 z[o]7 [ole lolol /eo[2[ely[a sf@ts [wo [el fs lols lo [3 Lo [as To afots fetal e fs TolaTo ols ols le slolwlols|m)~»lolsla tas -@ [a To HINTS AND SOLUTIONS L @ @ © @ oO) © Task | can not be assigned to either person 1 or 2i.. there are 4 options, Task? can be assigned to 3 or4 So, there are only 2 options for task So required no. of ways ~ 2 options for task 2 % 3 ‘options for task 1 * 4 options for task 3 * 3 options for task 4 2 options for task S = | option for task 6 Dx3MAK IID = 14d Therequired number of ways (1040 +17 +)-1 =879. ‘Total number of words that can be formed =10", Number (of words in which no letter is repeated = !P,, So, ‘number of words in which at least one letter is repeated =10°— "P,=69760, No. of words starting with Aare 4 !=24 No, of words starting with Hare !~24 No, of words starting with L are 4 !=24 These account for 72 words Next word is RAHLU and the 74 word RAHUL: Two pairs of identical leters can be arranged in 'C at ways, Two identical leters and (wo different letters can be arranged in °C, different letters can be arranged in *P, ways Total no, ofarrangements Fi 8 ma. a Let there be porticipants in the beginnin number of games played by ¢ 2484, Then the 2) players = +6=84 (Two players played three games each) = (0-2)in~3) = 156 Si Sn-150=0 S n= 1 Total number of arrangements of letters in the word GARDEN = 6!=720 there aretwo vowelsA and E,in halforthe arrangements proceeds E andother half A 0. 16, ) wo © © @ @ o) o) © Six balls ean be selected in the fellowing ways: onered ballsand 5 blue balls or Two red balls and blue balls Total number of ws IC, x 1C,+30 x7C Dar Sxaxt 63+ 105168 Two possibilities are there @ Chemisty part I is available in 8 books with Chemistry pat UL Chemistry part Il is available in 8 books but Chemistry part lis not available, Total No, of ways a 1x8¢,+7C 14645 6+ 3x2 Required number of ways=ways of selecting 4 objects out of 6 given objects 35 1 Iisa question ofarrangement without repetitions. Required no. of ways = 54 3—60 Asall the pointsare equally spaced, the area ofall the convex pentagons will be the same. No. of words which have = tatal no, of words: letter is rep 105, = 100000109 «8x76 09000 - 30240 = 69760 Required sum=(2+4+6+ 100)-(00++ 20... 100), No.of ays in which 6 men ca table=(6- 1}t Now women can be arranged in 6! ways. Total Number of ways = 6!» 5! Two particular girls can be arranged in 2! ways and remaining 10 girls can be arranged in 10! ways Roquired no. of ways= 2! « 10! least one letter repeated total no. of words in which no ated +100)+(5-+10+ 15+ 2580-+ 1050-530 =3070, be arranged ata round Permutation, Combination & Probability and itit Sothere avoidinga triangle. 19, (@) Requiredno. of the ways c,x4C 20, (@) Total number of possible routes from the city A to city C=4x6 4 21, (@ Probability that only husband is selected 1 rt Probability that only wifes sel PUDPOP), Probability that only one of them is selected 46 10 2, (@) Total of seven ean be obiained in the following 1,11,4 in 3 =4 ways {a Similarly Hence, required probability [/- Exhaustive no, of eases 666% 6 64] 28, (a) The faulty machines can be identified in wo testsonly ‘if both the tested machines ure either all defective or following tree di all non-defective, See th L Reqd. Probability = v3 The probability that first machine is defective (or non-defective) is and the probability that second 1 ‘machine is also defective (or non - defective) is = as 1 defective machine remains in total three machines. 24, (©The total number of ways in which 3 inte chosen from first 20 integers is The product of three inte (© Sietters out of 15 letters can be selected in SC, ways, 18, (@ Toconstruct2 roads, three towns can be sclected out fd in 4 x3%2 = 24 ways. Now ifthe third road goes fiom the third own to the firsttawn, a tiangle's formed, 24 ways to form a triangle and 24 ways of are four objects, thre repeated) cts will he even ifat least cone of the integers is even. Therfore, the required %, 0, Ast probability= 1 —Prob. that none ofthe three integers [Three odd integers can be chosen in thereare 10 even and 10 odd integers. (@) Ifa probability p is assigned to each even number, then 2p is the probability tobe assigned to each add number which gives 2 x3+ px 3 =9y 1 pet 9 P(E) =Probabiltyor wating 4, Sor6 12.14 99 9 (0) The condition implics that the last dig in both the integersshould be, 1, $or6and te probability Pla io) 100” 25 [ ~ Thesquares of numbers ending in 0 oF I oF Sor 6 also 0 or Lor 5 or 6 respectively] (@)_ Seven people can seat themselves at a round table in 6! ways. The number of ways in which two listingnishssl persons will eenext to each other=2 (8) 1, Hence, the required probability 63 ways The number of ways of selecting those pairs which havea sidein common (2) 4) (gn2erges soa) 112 [Since each of the commer squares has twos! cach of 24 squares in border rows, ther than corner nes has three neighbours and each ofthe remaining $36 squreshave four neighoursandin this computation, eat pair of squares has been considered twice] H = 4 ence eguired probability = (b) A and B will contradict cach otherifone of the events AB" OF A'rB occurs. The peobability of this happening is AUANBIU(ANA)= Pld B+ PLAN) PA) PUB) + PCAYP CB), because A andl are independent, Therefore, putting AAA) = 0.7 and PCB) = 0.8 the required probability is (07)(02) +(03)(0.8)=0.38. (0) The probability that 4 cannot solve the problem AB a.) 2. (a) R © o The probability that B cannot solye the problem a4 The probability tha both A and B cannot solve the probiem bh ra The probability hat at east one of 4 and 8 can Lou sve the problem " 12 12 u The probability thatthe problem is solved = [> Probability (sending a correct programme) Probability (the packet is not damaged) =1 Probability (there is no short shipment) = pays wo Total number of halls=5+7~8=20 Probability thatthe first ball drawn is white Roquired probability = 4.1 alls are drawn with replacment, all the four events will have equal probability, Therefore, required probability ria PES a" ise Require probability Probability that bal from bag 4 is re and both the balls from bag 8 are black + Probability that ball from bag is black and one black and one red balls are drawn from bag B 5G Gg GG 5c, tog os 8s 1S Total no, of outcomes when two dice are thrown = n (S)~ 36.and the possible eases for the event that the sum of numbers on two dice isa prime number, are 0,0,0,2),0,9,0,0,2,0,2.3,2,5,8,.2.8.4.4, DGG, INSOVG 146.5) Number of outcomes favousing the event agay= 15 Roquired probability 36 @ @ © © © Permutation, Combination & Probability The probability of selecting a bag Now, probability of wetting a white ball from the frst 133 bag and probability of getting a white ball from thesecond 1 re 6 Required Probability = The probability that a white ball is drawn either from the first or the second ba balls can be drawn in the following ways I red and | green or 2 ret or 2 green Required probability The event that the fifth toss results a head is independent of the event that the first four tosses results tails Probability of therequired event If six coinsaretossed, then the otal no. of outcomes 64 Now, probability of getting no tail Probability 0 ting atleast one tail 0000 1234 n()=4x3x2%1 n(Ay=3x2%2x1 Required probability = Total no, of numbers 4 nS)= MC, The product of four numbers could be positive when, (@) all the four numbers chosen are positive or (0) all the four numbers chosen are negative or (6) two.f the chosen numbers are positive and two arenegative. 6 positive Required Prob,= Daa inerpretation questions are based on he information given ins the iables.and graphs (Classification of Data Interpretati — confi soder ad vk Gay i Gap ° TABLES A table isone of the easiest way for summarising data A statistical tableis the logical listing of rclated quantitative data in vertical columns and horizontal rowsof numbers with sulicient explanatory and qualifying words, phrases and state form of titles, heading and notes to make clear the meaning of dara, ents in the EIB REMEMBER Sum of all items Total number of tems 2.=The bars aredrawn proportional in ength tothe total and then divided in the ratios oftheir components % change (increase or decrease) Average 100 GRAPHS Graphs area convenient way torepresent information, The graphs should be labelled properly to show what part of the graphs shows what a value. 1. Bar Graph - Bar diagram consists of a number of equidistant rectangles. One foreach category of the data Data Interpretation in which the magnitudes are represented by the length or hight of rectangle, whereas width of rectangles are immaterial, Thus, a bars just onedimensional as only the length ofthe bar is tobe considered and not the width. All the bars drawn in adiagram are generally ofuniform width ‘which depends on the number of bars to be constructed and the availability ofthe space, Types of Bar Graphs are @ Simple Bar Graph : Weis used to represent only one ‘dependent variable. (i) Sub-ivided Bar Graphs: These the break down of. total into its component bars, A bar isivided into different seuments, each segment respresonts 2 given component. Different shades, colours, designs etc. are used to distinguish the nt the various components. To compare, the order of reused torepresent various components. An indx is given to repres ‘various components in the different bars is same. (ii) Mutipte Bar Graph (MBG) + Wher. combination of inter-rolaved variables are to be represented graphically, multiple bar diagrams are used. These are extended form ofsimple bar diagrams. In M.B.G. many aspects of the data are presented simultancously with separated bars o various shades ‘flours, An index is given to explain the shades or colours used, Line Graph(L.G) : LG are used to show how a quantity changes, very ofen the quantity is measured as time changes. I the line goes up, the quantity is inereasin and the line goes down, the quantity is decreasing. Ifthe line is horizontal, the quantity is not changing Pie Graph PG) : isa pictorial representation of numerical data bynon- intersecting adjacent sectors ofa circle sector's area of cach sector is proportional tothe magnitude of the data represented by the sector 360 3.6 1% of total value 100 The % of components parts can be converted to degrees by multiplying 3.6 Degree of uny component part component valve total value 360. Data Interpretation stlons (Qs. 1-20) : Study the fallowing andanswer the given questions. jon carefully Number of players participating in thrce different games| different schools. [Number of players arieiating i three Aiferent games i fe afferent Schou ic = : tsk ia BKto Kno AAA School School? Schools School Schooks Sehunts 11, What isthe total number of players participating in hockey fromall the five schools together? @ 24 @) 2 ( WM) mw 12, What is the respective ratio between number of players participating in basekotbal from school-Landithe number of players participating in Kho-Kho from school-3? (@ 5:7 0) 7:9 (©) TS Md) 97 13, In which school is the number of players participatin hockey and basketball together second lowest? (@) School -1 (b) School -2 (©) School -3 (School -4 14, Number of playerspartieipating in Kho-Kho from school-4 is what percent of number of players participating in hockey from school-2? @ 2 8 © % WH 15. 25% of the number of the players particpating in hockey fom School-$ are females. What is thenumber ofthe hockey players who are males in school-5? @ hb © 8 © 0 @ 7 Directions (Qs.66-70): Study the flowing bar-graph carefully andanswer the followingquestions. Earnings (in rupees) of three afferent ‘persons on four diferent days [Narcan Ga Rahat ‘Thursday Wednesday T Tuesday Monday === 0 1 om DD 500 16, ASS What is Gita's average earnings over all the days together? (285) () ©290) (©) 3320) (@) 7310 ‘What is the total amount earned by Rahul and Naveen together on Tuestay and Thursday together? () 1.0408 () F100. © 2980" (@) 2940) Gita donated her earnings of Wednesday to Naveen, What -arnings on Wedlneslay after Gita's ‘was Naveen’s total donation? (520% () %50 (e540 (@) 500) ‘What isthe difference between Rahul’ earnings on Monday and Gita’s carnings on Tuesday? @ 240: ) %20% (©) 50% (30% Whats the respective ratio betwoen Naveen's earnings on Monday, Wednesday and Thursday? 3 () 8:6:5 @ 7:3: (@) 8:7:4 @) 93:4 Directions (Qs. 21-25) : Study the following pie-chart and answer Percentage wise distribution of emplayeesin six different professions Total number of employees = 26800 ‘Whatis the difference between the tral number of employees in teaching and medical profession together and the number cof employess in management profession? 670 (©) 7700 © m0 «@ 70 In management profession three-tourth of the number of ‘employees are female. What isthe number of male employees in management profession? @ 9 () 13 © 1156 @ 169 25% ofemployees from film production profession went on a strike. What is the number of employees from film production who have not partiipated in the strike @ 27 () 3819 © 38 @ 13 £86 24, What is the total number of employees in engineering profession and industries together? @) sos () Sse (© 567 (58% Directions (Q526-30): Monthly Bill (in rupees) of landline phone, electricity, laundry an months, Study the tab carefully to answer the questi Data Interpretation In teaching profession ifthrce-fifth of the teachers are not ppermancat, what is the number of permanent teachers in the teaching profession? 140s, 1764 6s i704 © © ©) @ that follow: nd mobile phone paid by three diiferent people in five different ‘Monthly Bills Month Electricity Laundry Mobile Phone Dev [Manu| Ravi] Dev [Manu] Ravi ] Dev [Manul Maren | 234 | 190 [ 113 | as [ 245 [ ais | os 6s | vas [238 [as April 124 [ 234 | 321 [270 | 220 [135 | ast 35 | 164 208 May 156 | 432 [211 | 86 [aso [9x | 232 132 [3 332 [June 7 [123 | 124 [124] 150 [ue | 213 isa [ 245 | 134 | 125 [iuty aa | os [ ise | 2as [tos [ist | aia | sa2 [vas [32a [ane [ se 26, What isthe total amount of ill paid by Devin themonth of 29, In which months respectively didl Manu pay the second June forall the four commodities? highest mobile phone bill and the lowest electricity bil? (a) 608 ) 763. (@) Apriland June ( te ) Fay (©) April and M: 27. What is theaverageclectricity bill paid by Manu ever all the (©) March and June fiye months together? (March and May (a) T1836 &) e149 30, What isthe respective ratio between the electricity bill paid (©) T1s9e ) 178 by Manu in the month of April and the mobile phone bill ‘What is the difference between the mobile phone bill paid by Ravi in the month of May and the laundary bill paid by Devin the month of March? (@ Fase (e198 B76 z 167) ) ©) Directions (Qs31 Study the table carefully to answer the paid by Ravi in the month of June? @ 7:49 () 27:65 © 4:9 (@) 1Bs:184 1estions that follow: [Station [Arrival time] Departure [Halt ime (im Name time nui [Dadar [Sarina pee] 1205 on_| Distance travelled | No. of Passengers boarding the train at each station Nadiad Jn. | 7.0ram 703m | 2unnutes VasaiRoad | 123iam | 12s¢am | Smmue a Surat 4.15 am | —420am | Sminutes Vadodara ‘Sminuiss 00k Ahmedabad | — 8.00 am 20 minutes 86 ka S40 pm Bhuj Fading pom 31. What is the distance travelled by the train from Surat to Nadiad Jn.? (@) 1764 () 188m (©) 183km @ 18km 32. How much time does the tain take to reach Ahmedabad after departing ftom Anand Jn. (including the halt time)? O77 kar the.s9min he he 45 min oO) ©) (©) Uhr 47min. o What is the respective ratio between the number of passengers boarding from Vasoi Read and ftom Abmodabad inthe train? @ 2:17 © a:9 1:9 15:13 ©) @ Data Interpretation MST HM. IP halt time (stopping time) of the rain at Vadodara is 38, Distance between which two station is second lowest? decreased by 2 minutes and increa minutes at (@)_- Nadiad Jn, Ahmedabad Ahmedabad. At what imewill the train reach Bhuj? (b) Anand J, to Nadiad (@) 6:10am (b) 6.01 pm (0) Dadarto Vassi Read (9) 60Sam () 6.50pm (8) AnandlJo, to Vadodara Directions (636-40 ble carefully oanswer the questions that follow: Temperature Month Bhuj | Syaey Ontario [Kabul [Bei Max. | Min. | Max Max. [ Min. | Max. | Min_ | i. i Sepienber uw] ey?[s;i[s«lale October 2 TPsftel[s] le I Novem 8 3[4)o[s |» [as 1" December, 9 2 3. A 7h 3! 3B 2 3 1 January 4 a 3 4 19 | 20 u Ss 3 3%. Whatisthedifference between the maximum temperatureof 38. In which month (on 1° day) is the difference between Ontarioon 1 November and the minimumtemperature of -—-™aXimum temperature and minimum temperature of Bhuj ae @ Tesqronter 1 Ocuber a se @ te Nowenbs (0) (Decne @ Isc @ ire 39, Whatistheaverage maximum temperature of Beijing over all 37. In which month respectively the maximum temperature of the months together? KabulissccondhighestandminimumtempersturcofSyuney —@) 84°C &) 96¢ ishighes’? (76 () 92% (6) it Cieiober and’ t*ianuary 40, What is the respective ratio between the minimum temperature of Beijing on 1* September and the maximum {): “Ul Dewber end 1° November temperature of Ontarion | October? (9 P* December one! 1* January (3:4 ) 3:5 (@) 1% September and 1* January © 4:5 @ us tt ots Tofu] [is Tm [21 To [26 To [31 Lo [36 Lo 2/6|]7/@/e2/o]7/o]2/ol7/olzlealy|o 3 Pots Ta 13 [Tm [as To [3 [im [8 To [3 To [8 To +t@ts Total @e]@ [se] Tolar [w [9 To 3 bot Tats Ta Te Tia Pos To Tae To @ La Toy INTS AND SOLUTIONS 1. (b) Difference between cost of | kg appeal east of | 4. (a) Totalcost of 4Skgs grapes from Hoshiarpur=45 » 190 guava in S cities. T8550 JF 160-60=100 D 130-9040 Aer discount 4% Ravinder paid=8550~ 8 C 180~120= 60 H %-30=60 zx208 R 40-20-20 5. (©) Costofl ke apples from Ropar Cost is second lowestin Delhi. Cost of The grapes from chandigarh 2 () Costoft kg guava in Jalandhar =® 60 40:90 Cost of2 kg grapesin chandigath =% 90 «2=% 180 4:90¢22:3? 6. @_Numberofstudents in university lin 2010= 20,000 ‘Number of students in university lin 2012~ 26,000 Difference = 20,00 -20,000 0 $8), Cos ene fe Bau 3>-130— 29%, (@) Number of students in university in 2007~ 10,000 Coat? Kay grace fe ans 925 30 521 Number of students in university lin 2011 30,000 Tefal cost that Ram pay=390-+ 18052570) Total stilents = 10,000 + 30,000 = 40,000 w op= 2 x190=33.3 = 34% ixo 0. 16. @ @ i} © 1} @ @ i} @ @ @ i) @ Total no, ofstudents in Uni No.of females students: n 2010=15,000 fof 15000 as 25.15, 000 oo No. ofmale students= 15,000~3750= 11250 Number ofstudenisin university in 2011 = 25,000 Number ofstudents in university lin 2010 20,000 25000-20000 = 5000 5000 20000 Ditference between Number of'students in university 1& Number of students in university I for the year 2007 1.000 20 Lg.000 209 L000 amo 5000 2o1L 000 m2 15000 Difference was highest in year 2012. No. of hockey players in all schools =68-+80+54-+50+36=288, No. of asket-ball playersin school-1 =42 No. of Kho-Kho players in school-3 = 48, Rutio=42:48=7:8 No. of Kho-Kho players in school-4 = 32, No. of hockey players in school? = 80 100= 40% key players in school-5 = 36 No.of females = 25%, No, of males = 75 No.ofmales = Gita's average earning over all the day's 100442042004 140 _ 1160 _ ¢ a9) 4 4 Rahul cared on Tuesiay and Thursday = 180+ 340=%520 Tetal= 520 +420=T940 Giradonated Naveen = €420 Naveen's total earning on Wesnestay 420-120-8540 Rahul's earningon Monday=%240 Gita's earning on Tuesday =% 200 Dill 240-200-740 Naveen's earning on Monday, Wednesday and Thrusday 380: 130: 180 =9:3:4 No. of employees in teaching and medical = 42% No.of employees in management ~17% Difference =42~17=25% 26800%25 _ gro 100 In management + employees are foal © crnployee ty rv 5 employees in management are male » @ c) @ @ © @ » @ @ o © @ i} Data Interpretation 268000171 aay 104 In film proxtuction emplayess on strik No. of employees notin the strike ~7 ‘No. of employees notin strike No, of male employees: 8001 100 Total No, ofemplayees in engineering industries 94 13=22% 26800 100 = 5806 th of the teachers are not permanent Zh ofthe teachers are permanent 26800215 1005 Total amount paid by Devin June for all commodities = 123+ 150+ 324+ 134 zi Averaged 1608 No. of permanent teachers = tricity bill paid byManu in all five months = 315+ 135+98+ 116-131 795 z Mobile phone bill paid by Ravi in May=2 143 Loundey bill paid by Dev in March =® 223 Diference 143=z 180 Electricitybill paid by Manu in April = 135 Mobile bill paid by Ravin June =245 Ratio= 135: 245=27:49 Distance travel by train from Surat to Nadiad Jn, 440-287 — 183 kms Total timetaken= Ihr. 3Smin+ 2min +2min + 20min = thr 59min Ratio between No. of passengers boarding from Vasa Roadand from Ahmedabad 378 :306=21 17 Total time increas: Train will each Bhuj at PM Distance between Anand Jn. to Vadodara is second lowest Difference between the max, temperature of Ontario cn Noy. andthe min, temperature of Bhuj on 1* Jan. =4-(7)= 11°C 5:40PM +21 min =6:01 Differencebewween max. and min. temperature of Bhuj Sept =10°C Oct = 1490 Nov.= 11°C Dee. =75C Jan. =3°C Average of max. temperature of Beijing over all the months= 43.= 86°C Ratio between minimum temperature of Beijing on 1 sept, ancl maximum temperature of Ontario on Pct. SrIse3:5 eeu e PaCen ne) Analogy The meaning ofanalogy is ‘similar properties‘or similarity Ifan 4, object or word o digit or activity shows any similarity with another object or word or digit or activity in terms of properties, type, shape, size, trait ete, then the particular similarity will be calledlanalogy. For example, cricket: ground and chess: table are the analogous pairs (why), In fact, both pairs of words have similar relationship in terms ofplace of playing as cricket is played in the ground and similarly chess is played on the table. In this chapter we will discuss different types of analogy because problemsbasedon analogyase an important category of questions tobe asked in almost all examinations of competitive love ‘TYPES OFANALOGY 5 T__ Tool & object based analogy: Thisestabishes arelationship betwen a tool and the object in which it works, Similar relations has to be discovered from answer choices Examples: Pencil Paper Pen Poper Scissors Cloth Saw Wood 6 Emser Paper Synonym based analogy : In such typeofanaloey two words have similar meaning. Examples: Big Large Huge Gigantic Endless Etemal Thi Sim Benevolent Kind Notion dea Huge Big Worker & tool based analogy: This establishes a relationship between a particular tool and the person of that particular profession who uses that tool Examples: Writer Pen Painter Brush Cricketer Bat Blacksmith Harmer Barber Scissors Hunter Gun Worker & product bas gives a relationship between a person of particular profession and his/her creations. alogy: This ype of analogy Examples: Batsman Run Writer Book Author Novel Singer Song Poet Poem Journalist News Causes & effect based analogy: In such type of analogy Ist word sets and the 2! word isthe effet ofthat action, Examples: Work Tirediness Bah Freshness Race Fatigue Shoot Kill Opposite retationship (Antonym) based analogy :Insuch Iype of analogy the two words of the question pair are opposite in meaning. Similar relations has tobe discovered fromthe answer choice word pairs, Examples: Poor Rich Fat Sin Tall Short Big Small Light Dark Avoid Meet n such type of analogy, one word ismasculine and another word is ferminine ofit In fct, itis a‘maleandfemale’ or ‘gender’ relationship, Exar Man Woman Boy Get Nephew Nice Bull Cow Duck Drake Ba 8 Classification based analogy: This ype ofanalogyishased 14 on biological, physical, chemical or any ether easitiation In such problems the 1* wordmayrhe elassified by the 2! Examples: Cow Animal Git Human Oxygen Gas Water Liquid Snake Reptile 1s Parra Bind Function based analogy : In such type of analogs, 2nd word deseribes the function of the I word Examples: Singer Sings General Commands Player Plays W. Surgeon Operates (Quantity and unit based analogy: In such type of analogy 2"! word isthe unit ofthe first word and vice-versa Examples: Distance Mile Mass Kilogram Length Meisr 5 Finished product & raw material based analogy :In such type of analogy the I word is the raw material and 22 ‘words the end product ofthat raw material and vice-versa. Examples: Yom Fabric Mik Cord Flour Bread Luex Rubber a Grape Wine Frit Juice Utility based analogy : Insueh type of analogy the 2! word shows the purpose of the 1" word ar vice-versa, Examples: Pen Writing Food Eating 19, Chair Sitting Bed Sleeping Bat Playing Symbolic relationship based analogy: In such type of analogy, the 1" word is the symbol of the 2! word and Examples: White Peace Red Danger Black Sorrow Red cross Hospital Swastika Fortune Analogy Adult & young one based analogy’: In such type of analogy, the 1* word is the adult oneand 2%! word is the you of the 1" word oF vico-vers Cow calf Human child Dog Puppy Duck Duckling Subject & specialist based analogy: In such typeof'analogy the 2"! word is the specialist of 1 word (subject) or vice Examples: Heart Cardiologist Skin Dermatologist Habit based analogy: In this ype of analogy 2"! word isthe habit of 1" and vice-versa, Examples: Cat Omnivorous Tiger Camiverous: Cow Herbiverous Goat Herbivorous Instrument and measurement based analogy: We soe in this type of analogy, the I¥ word is the instrument to measure the 2 word and vice-versy Humidity Barometer Thermameter T Sphyemomanemeter Blood pressure vidual & group based analogy : Second word is the sroup of 1* word (oF vice-versa) in such type oPanalogy Examples: Cow Herd Sheep Flack Grapes Bunch Singer Chorus State & capital based analogy: 1* word s the stateand 2 Word is the capital of that state (1* word) (or vice-versa) in the analogy like this, Paina West Bengal Kolkata Maharashtra Mumbai Karnataka Bangluru Note: Analogy bases! on country and capital is very similar tothis type of analogyin which weput name ofthe country in place ofthe name of state and country capital in place of state capital. For example India: New Delhi and Nepal: Kathmandu, Analogy 20. Analogy based on individual & dwel ws place : In sich type of analogy I wordis the individual & 2"! word isthe dwelling place ofthat individual (1* word) and vice-versa Examples: Horse Stable Bee Apiary Dog Kennel Birds Aviary Monk Monastery Human House 21, Analogy based on worker and working place: In this ype of analogy the |S word represents person of particular profession and 2° word represents the working place of that person (1% word) and vice-versa Examples Dostor Hospital Clark Oitfice Cook Kitchen Professor College Teacher School 22. Analogy based on topic study: 201 word (or vice-versa) inthe analogy like this. ‘word is the study ofthe Examples: Birds Omithology arth quakes Seismology Eggs Zoology 2. Analogy baved on letters (or meaningless words) Case I: (Forward alphabetical sequence) Examples: (CD: FG::PQ:UV Here CD and FG are in the natural alphabetical sequence. Similarly, PQ & UV are inthe natural alphabetical soquence. Case Hs (Backward ot opposite alphabetical sequence) QP: VU In fact this case is opposite of ease 1 Case IE: (Vowel Example ATL: VX: TPR: ORS. Here, the 1 wo words start with the I two vowels A & E consonant relation) and the next two words start with the next two vowels I & O. Last two letter of every word are consonants. Case 1V: Example Skip letter relation) ABC: FGH:: UK : NOP Here betwoen ABC & FGH two letters skip and they are D& E, Similarly, between IJK & NOP wo leters skip and they arcL &M, Case V: Jumble letters relation) Example: @ LAIN: NAIL EVOL: Love Here the 1terin gets reveresed to produce the 2"! term and similar elation is shown in between 3 3% and 4"Mterm, ABCD: OPQR:: WXYZ: KLMN In i each letter ofthe 1* group ‘ABCD is moved fourteen. steps forward to obtain the corresponciing letter of the group OPQR’.A similar relation is established betwen the third group ‘WXYZ" andthe fourth group “KLMN NOTE Every (ype of analogy discussed tn may have different variations of problems and you can ges perfection on them by proper practice ony Format ofthe questions EIN | : Lion is to fleshas cow is to @ Sol. snake () grass worm (@) animal Lion eats flesh, similarly cow eats grass. Hence option (b) is the right answer: EEE 2 : Pen: Writer @ © Sol. Batsman Brush © Fi Stick (@ Bat Option (d) isthe correctanswer because a writer uses pen to-write and similarly batsman uses bat to play. [EMM 3: NcvP- oDEQ @ © term will goon MPRO LOQN (b) NQOL, QM (® QNOL |. Option (a) is thecorrect answeras leters of term govone step forward tobe the 2! term. Similar! the leners of #2 tcp forward to be the 4" term (Letters of step go one step backward to be the 3" term). EME 4 : Bulky: Fat @ © Sol. (0) Happiness: ? © Usly (@ Somow Bad Joy is the correct eption because “Bulky” isthe synonym of Far and similarly “Happiness” is the synonym ‘joy Now, we can say that we have discussed almost all type of analogy to be asked frequently in the ‘examinations surprise kind of problems while solving the problems under this segment, But by practicing more & mors, you can be master in solving these problems. Only keop in mind the flowing: But examinees must prepare for any (1) Youmust have strong word power 2) You must have good understanding & reasoning ability. G) Youmust have good general knowledge. v Analogy Gene ER QERCISE Which ofthe following hasthe same relationship as that of Money : Wealth (a) Pity: Kindness (©) Cruel: Anger (©) Wise: Education (@)_Pride: Humility Which of the following is related to ‘Melody’ in the same way as ‘Deligious' is related to“Taste"? @) Memory (©) Highness (©) Voice (@) Speak Ina cartain way Diploma’ isrelated to"Education’, Which ofthe fllowingis related to Trophy’ in a similar way? (a). Sports () Athlete (c) Winning (@) Pree In acertain code CHEMISTRY is written as NFIDITUSZ. How is BEANSTOCK writen in that code? (a) CFBOSLDPU (b) CFBOSUPDL (©) OBECSUPDL (@)_ OBFCSLDPU Which ofthe following does as that of Cloth : Garments? (a) Leather : Footwear (6) Wood: Furniture (o) Earthen pots:Clay —(d) Gold: Or Clock is related to "Time" inthe same way as “Vehicles related to which ofthe following? (a) Driver () Read (c), Passenger (@ Journey Lllness” is related to“Cure” in the same way as “Griet”is related to (a) Happiness (0) Ecstasy (e) Remedy (@)_Solicitude at havethe sume relationship ‘Necklace’ isrelatedio ‘Jewellery in thesame wayas* Shin” is related to (a) Clo (©) Cotton (6) Apparel (@) Thread Bouquet’ is related to ‘Flowers’ in the same way as ‘sentence’ is related to Letters (©) Paragraph (©) Content (Words Which ofthe followi sto FLOWER in the same way as RTERBN relates t SECTOR? (a) RWLGPE (b) FOFKUQ (c) EOPMXS (@ RWLEND Which pair ofthe letters in the word BEAUTIFUL has the same relationship between its letters with respect to their position inthe English alphabet as the pair EA in that were has beween its leters? 1B LF (©) @ Al Income’ is related“ Poi’ in the same way as ‘Expenditure’ is related to (a) Sale 0) Receipts (c) Surplus (Loss Electricity’ is related to“ Wire’ the same wayas "Water" is related to fa) Botle ©) Jug (©) River (@) Pipe ‘Hospital’ is related to Nurse’ in thesame way as ‘Court’ istelatedto (a) Justice (b) Lawyer (©) Judgement (a) Trial [By following certain logic “THEIR is written as. “TRIE” and *SOLDIER’ is written “SROLIED”, How is CUSTOM \writen in that logic? (a) UTSOMC (b) CTSUOM (© CUTsoM (@) YUsoMc ns : In each of the following questions, there are two Words set of letters / numbers tothe left ofthe sign: which are connected in some way. The same relationship obtains between the third words / set of letters / numb sand one of the four alternatives under i. Find the correct alternative in each question, Flying : Bird:: Creeping :? (a) Acroplane (b) Snail (fo) Ground (@) Flower 17, Check: Times: Thermometer: ? (a) Heat (b) Radiation (©) Encwgy (@ Temperature Man: Walk: Fish :? (@) Swim (b) Est (©) Live (@) Sleep Import: Export: Expenditure? (a). Defi (6) Income (oe) Debe (Tax Ovean: Water: Glacier:? (a) Refiigerator () lee (©) Mountain @ Cave PRIN: XZTV:: ILFH:? (a) NPRT (NPT ()_ NTRP (INP DRIVEN : EIDRVN BEGUM: (@ FUBGM (b) MGBEL (©) BGMEU (©) UEBGM 14:9::26:2 @ 2 ob © 1s @ a ACE) : OUZJ::SUXB:? (a) GXSA () Giz (9 cKPY (@ GMRB 6:24:52? @ 23 22 (©) 26 @ 20 Medicine: Sickness : Book: (@ Ignorance (b) Knowledge (©) Author (Teacher Bank : River : Coast :”” (@) Flood () Waves (Sea (@) Beach Supervisor: Worker (@) Junior: Senior (b) Elder: Younger (©) Deblor: Creditor (d)_ Officer: Clerk Analogy 1 3. Thun: Rain: Night 4H. CG:EI:F) (a) Day Desk ia) iM wu (©) Darness (@) Evening @ GK om ACE: IIL: MOO:? 41. Ovean: Pasi lnd:? ia) XVI wy 1x (@ Gisenland (by reland () Vix @ TUX @) Neerland (@) Borneo 31, NUMBER-UNBMER::GhOsT:? 12, Tuberculosis: Langs :Catarct:? (a) HoGst (b) Hocrs i Throw (0) Gast (a) Hesor © Sin i ty 2 Merrie? 8. Dasa: Sond’? @) 29 wo @) Rie ) sa 2 21 ©. Water @ Waves 33. Courts Jasice: School: ? 4M. Major: Baton: Colonel? (@) Teacher (b) Student (a) Company (b)- Regiment (©) Ignornce (8) Eduestion © Amy (6) Soldes 3a Clone: Drizle? 48. Stout: Whisper Run? tng (6) Sorm (a) Sty (tana (©) Flood ( Downpou © Wak (Hop 3S. Oxygen: Bum :Carbonoxie:? 4, TOMATO: MTOOTA::123412:7 (a) sole ‘b) Fam 314 ib) 6 (© Exdinguish (@) Explode © mua @ 2a 94 Teheran stan: Being: ? a Baste 200 (a) China (0) Japan a 3 2 9 Tukey (0) Males @ 2 a ws S082 40 ws 10 8 (100 @ @ 4 @o 38, Disease Pathology: Planet:? w Bevs23t fe) Astrology 1b) Geoto 2 2 ©) Aurenony (@)Palaortoog © @ % 29. Foresight: Anticipation: Insomnia? Sri t68 9 (o) Treatment (0) Disease (a) 24 26 {© Sleplewnes ——(@) Unrest 8 » tle, lo, oa, _o, 1] @ 2 Lele leo|2>_ol»|o} ele spe ps pets pets pols be ee ea ae oy @ [ie Tm [26 Ta (a) @ ol l@ 2 Te @ [#7 To Fs © | Te s Le |» Tm [2s To @ | | 10 | @ | 20 Ty @ wT so | a INTS AND SOLUTIONS 1. (a) They are synonymous 2 (e) ‘Delicious"is theadjectiveused fr “Taste’, Similaely, Melodious’ is the adjective used for “Voice” © 3. (@)_ A successful finish of ‘Education’ equips one with ‘Diploma’. Similary, « successful finish in “Sports” 6 (@) equips one with *Trophy’ 1 @ 4. (©) Reverse the frst four letters ofthe given word. Now, ullleters are code asone place fora urd asin English alphabet except the middle letter, which remains unchanged Ineach pair 2nd thing is madeup of Istthing while in option (¢), the Ist thing is made up of 2nd thing. The lock makes a journey of time. Cure ensures removal of illness in the same way as remedy insures removal of grief Bs & 0 2 @ » @ © nw B® a 1 (@) % 1 @ i @) wb) a a. W 2 a wo 4. B® 26. (a) a) Jewellery consists of Necklace ie Necklace’ isa kind of Jewellery’ Similarly, ‘Shirt isa kind of* Apparel Bouquet is bunch of flowers. Similarly, Sentence is set of words that is complete in itself. From SECTOR TO RTERBN; The second letter becomes thied, fourth becomes second, becomes first, Also, after subiracti the first, we get fourth, fom third, weget fi and fram fifth we get las and last ne letter from BEAUTIFUL a ‘When Income is morethan expenditure, it bears Profit ‘But when Expenditure ismore than income, then loss is the mediumto transmit Electricity. Similarly Pipe isthe medium to carry Water He Words are arranged in alphabetical order but fiom right to left I’becomes UTSOMC. As ‘Bird’ flies, in the same way, ‘snails’ creeps the first isthe working place ofthe savond First isan instrument to measure the second Asa man covers some distance after walking, in the same way, a fish covers some distance after swimming Hence the correct answer is (a) The words in each pair areantonyns. First consists of the second, As Similac 84x jar r—4z wu rr FN nov nop Fifth and third letters of the frst term are frst and second letters ofthe second term and first two letters ofthe first termarethird and fourth lttersofthe second The relationship is (2x4) x As, Similarly, aHyo stig c—%u uae py oR joy Bop Socond term =4» First term Fourth term=d = Third verm As medicine cures sickness, in the same way, books Bank is the land beside a river Similarly, coast isthe land beside a sea 0, 41 mM 4 4% @ @ © @ @ @ © o © @ @ @ » @ Analogy As supervisor supervises the worker, in the same way officer supervises the clerk As ‘Rain’ is followed by “Thunder Darkness’ is followed by ‘Night’ similarly As Similarly, Abou Mot cor M81 ESL Qtox First wo letters ofthe first ermarein reverse order in the second term and so are the next two let 11: 17 alternate prime number (skipping 13) 19: 29 alternate prime number (skipping 23) Firsts the place where thesecond is caparted, Sevond is more intense than the first. ‘Oxygen’ helps in burning while “carbon dioxide’ extinguishes fre, Teheran’ is the capital of ‘Iran’ and “Beijing” is the capital of ‘Chine’ (Firstiermy? Fourth term =(Thire tem Diseases are studied under Pathology Sesondtcem Similarly, planets are studied in Astronomy. The words in cach pair are synonyms, As, Similarly C248 Fou 61 JoL The largest ocean is Pacitie Ocean, Similarly, the largest island is Greenland. Tuberculosis is a disease of lungs. Similarly, Cataract isa disease oF eyes. ‘Oasis’ isa water pool amidst sand Similarly, island isa piece of land amidst water Remember : ‘Sea" would have boen theanswer ifwe had “desert in place of sand AAs Major heads a battalion, the Colonel commands a regiment Whisper is of Lesser intense than shouting, so is walking torunning, Substitute numbers for letters T-1,0-2,M-3andA~—4, 12=3243, 20= 3=22 B=] +1 Band 19 areprimes with 17 leftoutin berween 48= 71122-1141 168 132-1 AMAUPTO | When we come to solve the reasoning part while preparing for any competitive examination of objective nature. We findthat the prablems based on classification are the very important seement, You can see such questions in every question paper and this is the reason why examiness are advised to be well aware of classification part of reasoning, In this chapter, efforts have been ‘made to make, examinees af various objective competitive ‘examinations, fully aware of reasoni ‘You must have in your mind that what does classification mean, Infact in classification we take out an element outof somegiven clerentsand the clement to be taken outs different fFom the rest, (of the elements in terms of common properties, shapes, sizes, types, nature, colours, traits ete. In this wi clements form 2 group and theclement that has ben taken out is not the member of that group as this single element does not possesses the common quality to be possessed by rest of the clements. For example, ifwe compare the elements like, lion, cow, tiger, panther, bear and wolf then we find that this isa group of animals, How clo we classify them? To understand this lt us see the presentation given below = based on classification the rest of the Lion Cow Tiger Panther Bear Wath L o4 { J Jf 4 Wild Domestic Wild wil, Wild Wid animal animal animal shims! anima animal Here, if we want to separate out one animal then definitely that ‘animal will be cow because cow is the only animal in the group which is a domestic animal. Rest of the animals (Lion, Tiger, Panther, Bear and Wol?) are wild animals. Hence rest ofthe animals (Lion, Tiger, Panther, Bear & Wolf) forma group of wild animals, Separating out the domestie animal (Cow). ‘Similarly, out of6 letters A, M,N, U.P &-Q, we will ake out and forma group ofS letters M, N, U, P & Q because out of given six lesiers only A isa vowel while rest ofthe letters form 3 group of consonants TYPES OF CLASSIFICATION : (@)_Letter/meaningless word based classification @ Me word based clasifiea @ Di (General knowledge based elassificat Now we will discuss these three types of classifications one by Classification (1) Letter/meaningless word based classification =~ Such classifications are based on letters of English alphabet. So many groups of leters are given in the question in which one group is different from remaining groups andl hence the different group will be ouranswer, EEE 1 : Find the oxkl word out ofthe following options (@ Por () WY (©) DEH @ UN Sol (a) PQ ro) UY y 2 letter gap 2 letir gap 3 leer gap As it is clear that except option (d) al the other options have letters gap between 2nd and third letters and the Ist rwo leters, are in consecutive order. While in case of option (d) Ist two letters are in consecutive order but thereis 3 letters gap between 2nd and third letter separating it out ofthe remaining group ofthe letters. Hence option (d) isthe eorreet option GE»? : them 3 form a group in terms of some similarity, Find out the ‘option which does not fit into that group. Following are given four eptions and out of (@) TMNO-ONML ——(b) PQRS-SROP (© UKL-LKH (@) UVWX-VUXw LUIMINjo|__jojINjie| Sol. @) ERREGREET PHO|R |S} __ SIR} ow 2) 1/23 [6 ala [2h nn [if | © SPREE yey) v eee @ Above presentation makes it lear that (d) docs not fit into the groups. Bs 2. Meaningful words based classification : In such type of classification we have to take odd word out of the group of meaningful words. EEILIDDp 3: whien one oF the following words is not a part of the group formed by remaining words, The remaining words form the group onthe bass of certain similarity (@) Name 0) Game (©) Fame (@) Shame Sol, (@) is the correct answer because this is the only word Which has Sleters while the remaining words have 4 eters and hence options (a), (b), & (c) form a group separating out option (4). EXAMPLE certain thing common and so they form a group. Find out the ‘word which one is nota part of that group. ‘Slim Out of the $ words given below, fbur have Gad (6) Greets @ Sol: Option (c) is the correct option because this is the only option which has two vowels while the other options have only one vowel. Let us see the following presentation: sD Tens a -g 3. Digit based classification :-Insuch typeaf classifications fen to find out ane number that is nots digits or numbers ar part ofthe group of remaining numbers, Gm» a) 120) 1s ©) 9@ 20 ind the old muriber out ol: Option (c) isthe correet option because this isan odd number ‘while all the other options are oven numbers, ED « similarity and Four of the following numbers have some nce they form a group, Find out the number ‘which does not suit in the group, () 8982 @ 7 @ Classification Sol.: Option (b) is the correct answer as except option (b) all other options start with 7 & end with 2, 4. General knowledge based classification :- Such classification is doncon the basis of our general knowledge. No doubts that this is a word based classification but Without having general knowledge this type of questions can not be solved, GAG 7: Find the odd man out (@) Patna o) (©) Madhya Pradesh Sol: Option (cis the correct answer because Madhya Pradesh is am Indian state while all other options are expitals of Indian states. Patna is the capital of Bihar: Mumbai is the capital of Maharashtra: Kolkata s the capital of West Bengal Mumbai (@) Banglura and Banghiru isthe eapital of Karnataka. incase of Madhya Pradesh (itis an Indian state), we ean say that it as its capital in Bhopal EEELTDGp 8 : which ofthe ftlowi the group formedby remaining fouranimals? @) Care) g animals does not fit into Dog (@) Octopus Sol: Option (d) is the correct option as this isthe only animal cut of given options which is « water animal, Rest of the options are land animals. EEUIZp 9 : Four ofthe following given options have some similarity and so they form a group separating ut one option. Find out that separate option (@)Tendutkar (©) Dravid (© Sania Mirza (8), YuorajSingh Sol. Option (isthe correct option because Sania Mirza isan Indian tennis player. Whilethe rest ofthe options are indian crieketers, Now this chaptcr has come toan end, Readers areadvised to move as per the following steps while solving the problems rela Sept: Se Step ed to classification the given options with a serious eye. e relation of similarity among the given options. Seep III: Find out the one word not having the common similarity like other four option and that one word will be your answer, Classification 6 2 ER QERCISE Four ofthe following five are alike ina certain wayand so form ageoup, Which isthe one that does not belong tothat group? f) 9 8% (© ur @ L Fourof the Following five ae alike in a ertain wayand so forma group. Which isthe one that does not belong to that group? fa) Crow (0) Valure (e) Bat (@)_ Ostrich Four ofthe followin forma group. Which isthe one that does nat belong to that fivenrealikein acertain wayand so group: (a) Food : Hun (&) Water: Thirst (6) Air: Suffocation (q)_ Talent: Education Four ofthe following five arealike in a certain wayand so forma group. Which isthe one that does not belong. to that rou? (a) Teacher () E (©) Architect @ Four of the following five are alike in a certain way and hence forma group. Which one of the followit Doctor belong to that group? f@) 126 o) 12 © 6 9 Four of the following five are alike in a certain way and hence form a group. Which one ofthe following is different from the group? 26 @) 20 (©) 360 @ 170 Four of the following five are alike in a certain way and hence foema group. Findthe ane whichis different fromthe other four, (a) Rice (&) Wheat (c) Barley (@) Mustard Four of the following five are alike in a certain way and hence froma group. Find the one which sdifferent from the other four, (a) Arrow (0) Sword (©) Knife (Axe Four ofthe following fivearealike in a eertain way and so forma group. Which isthe one that dees not belong to the group? (a) 169 @) 179 B @ 49 Four ofthe following five arealike ina certain wayand so forma group. Which isthe one that does not belong tothe group"? (a) Listen o (©) Think @ Fel Sing 15 19, 2. Four of the following five are alike in a certain way and so forma group. Which is theone that does nat belongtto that group? (@) Jowar (b) Wheat (©) Mustard (@ Baia Four of the following fiven forma group. Which is theone that does not belong to that roup? (@) Naume () Size @ 1 (Shape Four of the following fiveare alike in a certain forma group. Which is theone that does nat belongtto that calike ina certain way and so vay and so group? f@ 2 w 2 (©) 12 @ 10 Four ofthe flowing fear alkn a certn wayandso fama gp, Which stheoe that does wa ielong oat a) Gime (©) Orange Ame th Ricolte susp tveatcatuas sad oer grap Wes ibeg a in way and so oes not belong to that (2) Aluminiam (©) Copper (9) Mercury @ rn Four of the following fiveare alike in a certain way and so forma group. Which is theone that does not belong to that group? @ w% 3 (9 @ 15 Four ofthe following fiveare alike in a certain way and so forma group. Which is theone that does not belong to that group? (a) May (b) December (© july (January Four of the following fiveare alike in a certain way and so form a group. Which his the one that does not belong, to that group? (@) lackal () Choctah (©) Tiger (@ Do Four ofthe following fv are alike in a eertin ayandso form a group. Which is the that doos not belong to that group? (a) Cheese (b) Butter (©) Mik @ Curd Four of the following fiveare alike in a certain way and so forma group. Which is theane that does not belong to that group? @ Bt ) 151 (o) Ist @ 1

You might also like